[MIX] Nierozwiązane problemy

Zadania z kółek matematycznych lub obozów przygotowujących do OM. Problemy z minionych olimpiad i konkursów matematycznych.
Regulamin forum
Wszystkie tematy znajdujące się w tym dziale powinny być tagowane tj. posiadać przedrostek postaci [Nierówności], [Planimetria], itp.. Temat może posiadać wiele różnych tagów. Nazwa tematu nie może składać się z samych tagów.
frej

[MIX] Nierozwiązane problemy

Post autor: frej »

[UWAGA] To zestawienie jest już zamknięte. W następnej wiadomości znajduje się aktualna lista nierozwiązanych problemów.

Witam.

Przez długie lata działalności forum, często pojawiały się problemy trudne (bądź łatwe ), które przez swoją złożoność zostały bez rozwiązania i po pewnym czasie zagubiły się w gąszczu postów. Celem tego tematu jest odświeżenie tych problemów i zachęcenie wszystkich użytkowników do zmierzenia się z nimi. Po kliknięciu na pogrubioną nazwę wątku, nastąpi przekierowanie do odpowiedniego tematu, tam można się zapoznać z dyskusją (o ile była) na temat problemu i ewentualnymi wskazówkami bądź próbami rozwiązania. Tam też można włączyć się do dyskusji na temat problemu, gdyż ten temat ma tylko funkcję informacyjną.

Problemy ułożone są chronologicznie (od najstarszego). Z tematów zawierających kilka zadań, w których niektóre zostały rozwiązane, zacytowałem tylko zadania nierozwiązane. Stan na dzień 22.08.2008, do tematu o numerze 80440 - nowsze tematy nie będą póki co dodawane na tą listę. Przy rozwiązanych problemach będę umieszczał napis: Problem rozwiązany przez użytkownika [nick_tego_który_rozwiązał] (przykłady poniżej )

Wielkie podziękowania dla kolegi mola_książkowego, który pomógł mi zbierać linki do trudniejszych problemów.

// Statystyki rozwiązań pojedynczych problemów (czyli nie liczę mixów i innych tematów, do których jest sam odnośnik) wedle widzimisię prowadzącego temat :
* 24.08, godzina 15:30 - zostało rozwiązanych 17 z pośród 54 problemów, szybko idzie
* 24.08, godzina 23:00 - 20/54
* 25.08, godzina 23:15 - 24/54
* 26.08, godzina 23:25 - 28/54 - w ekspresowym tempie połowa zadań się poddała
* 27.08, godzina 21:08 - 30/54
* 28.08, godzina 16:00 - 31/54
* 29.08, godzina 00:30 - 33/54
* 29.08, godzina 13:00 - 35/54
* 05.09, godzina 14:40 - 37/54
* 07.09, godzina 10:00 - 38/54
* 09.09, godzina 07:00 - 39/54
* 15.09, godzina 16:00 - 40/54 - ostatnich 14 nie(w pełni)rozwiązanych problemów "wylatuje" z tej listy z pierwszego postu i pojawi się w nowym zestawieniu
//

Nie przedłużając:

  1. Interesująca nierówność
    Problem rozwiązany przez użytkownika chris139
  2. Szukany ciąg
    Problem rozwiązany przez użytkownika przemk20
  3. Zadanie o czworokącie
    Problem rozwiązany przez użytkownika Menda
  4. "Kupowanie" ciągu Fibonacciego
    Problem rozwiązany przez użytkownika mol_ksiazkowy
  5. Znajdź takie piątki liczb pierwszych
    Problem rozwiązany przez użytkownika alchemik
  6. Konik na szesnastopolowej szachownicy
    Problem rozwiązany przez użytkownika Sylwek
  7. Wykaż jak najprościej...
    Problem rozwiązany przez użytkownika Lider_M
  8. Zadanie nie-"podobne" do innych...
    Problem rozwiązany przez użytkowników gryzzly92 i Sylwek
  9. Nierówność trygonometryczna
    Problem rozwiązany przez użytkownika Wasilewski
  10. Szukane wielomiany
    Problem rozwiązany przez użytkownika robin5hood
  11. Paskudne maksimum
    Problem rozwiązany przez użytkownika Sylwek
  12. Znajdz wszystkie takie funkcje rzeczywiste, że:
    Problem rozwiązany przez użytkowników polskimisiek i andkom (pośrednio )
  13. Intrygujacy ciąg - udowodnić zbieżność i obliczyć granicę
    Problem rozwiązany przez użytkowników robin5hood i limes123
  14. Rodzina podzbiorów
    Problem rozwiązany przez użytkownika mol_ksiazkowy
  15. Mocnia
    Problem rozwiązany przez użytkowników limes123 i Sylwek
  16. Pary
    Problem rozwiązany przez użytkowników Sylwek i mol_ksiazkowy
  17. Okresowość
    Problem rozwiązany przez użytkownika max
  18. Funkcja
    Problem rozwiązany przez użytkownika mol_ksiazkowy
  19. Trzy zadania
    3. Problem rozwiązany przez użytkownika Sylwek
  20. Szyfry
    Problem rozwiązany przez użytkownika Sylwek
  21. Liczba podzielna przez każde cykliczne przestawienie cyfr
    Problem rozwiązany przez użytkownika mol_ksiazkowy
  22. Puzle
    Problem rozwiązany przez użytkownika Sylwek
  23. Przyrównanie
    Problem rozwiązany przez użytkownika Sylwek
  24. Pseudosrodki
    Problem rozwiązany przez użytkownika Sylwek
  25. Rozwiązać równanie w liczbach naturalnych
    Problem rozwiązany przez użytkownika Sylwek
  26. Klopotliwy wielomian
    Problem rozwiązany przez użytkownika mol_ksiazkowy
  27. Znajdź liczby, nierówność funkcyjna
    Problem rozwiązany przez użytkownika andkom
  28. Perfidna tożsamość
    Problem rozwiązany przez użytkownika Sylwek
  29. Największa liczba rzeczywista
    Problem rozwiązany przez użytkownika Sylwek
  30. Środkowe a obwód
    Problem rozwiązany przez użytkownika Sylwek
  31. Liczba szczęśliwa
    Problem rozwiązany przez użytkownika mol_ksiazkowy
  32. Stała suma
    Problem rozwiązany przez użytkownika mol_ksiazkowy
  33. Reszta
    Problem rozwiązany przez użytkownika mol_ksiazkowy
  34. 2 równania funkcyjne
    1. Problem rozwiązany przez użytkownika andkom
  35. Trójkąty i rozkład
    Problem rozwiązany przez użytkownika Sylwek
  36. Funkcja i wielomian
    Problem rozwiązany przez użytkownika Sylwek
  37. Styczne + czworokąt w okręgu
    Problem rozwiązany przez użytkownika limes123
  38. Równanie Hurwitza
    Problem rozwiązany przez użytkowników Sylwek i mol_ksiazkowy
  39. Kilka zadań
    Problem rozwiązany przez użytkownika limes123
  40. 2 ciekawe zadania z geometrii
    Problem rozwiązany przez użytkownika limes123



Powodzenia!
Ostatnio zmieniony 3 lis 2008, o 21:16 przez Sylwek, łącznie zmieniany 5 razy.
frej

[MIX] Nierozwiązane problemy

Post autor: frej »

Witam.

Chciałbym zaprezentować drugie (na razie definitywnie ostatnie) zestawienie nierozwiązanych problemów.

Na wstępie kolejne ogromne podziękowania dla mola_książkowego, który przejrzał dużą część bardziej interesujących działów na forum w poszukiwaniu trudnych problemów (teraz można powiedzieć, że w przybliżeniu wszystkie ciekawe problemy zostały przez niego "wyłowione"). Lista się składa na chwilę obecną z 69 problemów w 66 tematach, które pojawią się po raz pierwszy w zestawieniu, a także 14 problemów w 13 tematach przeniesionych tu z pierwszego posta, które nie doczekały się przez prawie cały miesiąc pełnego rozwiązania (co świadczy, że nie są one zbyt proste :) ) - te ostatnie mają numery od 67. (czyli są poza kolejnością chronologiczną). Tematy zawierające najwięcej nierozwiązanych problemów umieściłem na samym końcu listy nie cytując ich treści.

Stan na dzień 16.09.2008, do tematu o numerze 82797. O ciekawych nierozwiązanych problemach z tego bądź innych działów, które nie zostały umieszczone na poniższej liście, proszę informować mnie przez: [usunięto link do nieistniejącego konta]. Przy rozwiązanych problemach będę umieszczał napis:
Problem rozwiązany przez użytkownika [nick_tego_który_rozwiązał]

##
Statystyki rozwiązań pojedynczych problemów (czyli nie liczę mixów i innych tematów, do których jest sam odnośnik) wedle widzimisię prowadzącego temat ;) :
* 16.09, godzina 18.00 - chwalebne pojawienie się nowego zestawienia :D , 0/69 + 0/14 ;)
* 17.09, godzina 00:00 - 23/69 + 0/14 - w 6h poszło 23 problemy, świetny start :D
* 18.09, godzina 21:30 - 27/69 + 0/14
* 29.09, godzina 20:00 - 32/69 + 0/14
* 30.09, godzina 00:15 - 34/69 + 0/14
* 05.10, godzina 01:00 - 35/69 + 0/14
* 06.10, godzina 22:00 - 37/69 + 1/14
* 12.10, godzina 16:00 - 38/69 + 1/14
* 29.10, godzina 20:00 - 41/69 + 1/14
* 03.11, godzina 21:00 - 41/69 + 2/14
* 04.11, godzina 17:30 - 42/69 + 2/14
* 23.11, godzina 14:00 - 45/69 + 2/14
* 24.11, godzina 22:30 - 47/69 + 2/14
* 07.12, godzina 22:20 - 49/69 + 2/14
* 13.12, godzina 21:00 - 50/69 + 2/14
* 16.12, godzina 16:00 - 51/69 + 2/14
* 30.12, godzina 16:00 - 52/69 + 2/14
* 04.12, godzina 16:30 - 52/69 + 3/14
* 25.01, godzina 20:00 - 53/69 + 3/14
* 27.01, godzina 22:00 - 54/69 + 3/14
* 28.01, godzina 23:00 - 55/69 + 3/14
* 08.02, godzina 13:30 - 56/69 + 3/14
* 03.03, godzina 20:00 - 57/69 + 3/14
powolutku do przodu ;)
##

Nie przedłużając:


  1. Sumy potęg dwójki
    Problem rozwiązany przez użytkownika Sylwek
  2. Szukany wielomian
    Problem rozwiązany przez użytkownika luka52
  3. Zadanie o czwarokącie
    Problem rozwiązany przez użytkownika limes123
  4. Suma kwadratów pól
    Problem rozwiązany przez użytkownika Sylwek
  5. Osie
    Problem rozwiązany przez użytkownika mol_ksiazkowy
  6. Planety
    Problem rozwiązany przez użytkownika Sylwek
  7. Przecięcia
    Problem rozwiązany przez użytkownika limes123
  8. Liczba trójkątna
    Problem rozwiązany przez użytkownika mdz
  9. Wybór punktów
    Problem rozwiązany przez użytkownika limes123
  10. Szukany wielomian
    Problem rozwiązany przez użytkownika Sylwek
  11. Udowodnij rowność z sumami
    Problem rozwiązany przez użytkownika Wasilewski
  12. Sprytne przejście
    Problem rozwiązany przez użytkownika Sylwek
  13. Podzielność wielomianów, współczynniki całkowite
    1. Problem rozwiązany przez użytkownika Sylwek

    2. Problem rozwiązany przez użytkownika Sylwek

    3. Problem rozwiązany przez użytkownika Sylwek
  14. Punkty na okręgu
    Problem rozwiązany przez użytkownika xiikzodz
  15. Kółka w kółku matematycznym
    Problem rozwiązany przez użytkowników exupery i Sylwek
  16. Kombinacja kosinusów
    Problem rozwiązany przez użytkownika mol_ksiazkowy
  17. Dwusieczna a boki
    Problem rozwiązany przez użytkownika limes123
  18. Wykaż istnienie trzech liczb
    Problem rozwiązany przez użytkownika limes123
  19. Szukamy okregu
    Dane mamy, w przestrzeni - dwie przecinające się proste a i b. Rozważamy wszystkie możliwe pary płaszczyzn \(\displaystyle{ \alpha , \ \beta}\) prostopadłe do siebie i t. ze \(\displaystyle{ a \subset \alpha \ \ b \subset \beta}\). Wykaż ze istnieje taki okrąg, że przez każdy jego punkt przechodzi prosta \(\displaystyle{ \alpha \cap \beta}\) dla pewnych \(\displaystyle{ \alpha \ \beta}\).
  20. Wykaż istnienie punktu
    Problem rozwiązany przez użytkownika luka52
  21. Turniej
    Problem rozwiązany przez użytkownika Sylwek
  22. Mantysa
    Trzeba dowieść, że jeśli liczba rzeczywista x jest postaci (*), to ciąg \(\displaystyle{ a_n}\) (**), nie jest zbieżny do zera...a czy może mieć inną granicę? Jeśli tak, to zbadać jaką:
    (*) \(\displaystyle{ x=\frac{k}{2^m}}\),
    (**) \(\displaystyle{ a_n=x2^n- [x2^n]}\)
  23. Rodzina
    Problem rozwiązany przez użytkownika przemk20
  24. Ciąg liczb dodatnich
    Wykaż fakt: istnieje stała \(\displaystyle{ \alpha}\) o tej własności, że dla każdego ciągu \(\displaystyle{ x_1,\ldots,x_n}\) liczb dodatnich, jeśli dla k>0 ilość wyrazów \(\displaystyle{ x_j}\) nie mniejszych od k, pomnożona przez k jest nie większa od m to :
    \(\displaystyle{ \frac{1}{m} \sum_{j=1}^{m} \log \ x_j \le \alpha}\)
  25. Prostopadłość przekątnych
    Problem rozwiązany przez użytkownika mol_ksiazkowy
  26. Trójkąt i okrąg opisany
    Problem rozwiązany przez użytkownika limes123
  27. Kolorowanie boków i przekątnych
    Problem rozwiązany przez użytkownika xiikzodz
  28. Liczba nierozkładalna
    Problem rozwiązany przez użytkowników przemk20 i andkom
  29. Nierówność z funkcją
    Problem rozwiązany przez użytkownika Dumel
  30. Sfera na czworościanie
    Dany jest pewien czworościan c, na którym opisano sferę \(\displaystyle{ s}\). \(\displaystyle{ \alpha, \beta, \gamma, \delta}\) są płaszczyznami stycznymi do tejże s, w odpowiednich wierzchołkach c , tj punktach A, B, C, D, przy czym \(\displaystyle{ \alpha \cap \beta=p}\), i \(\displaystyle{ \gamma \cap \delta=q}\). Wykaż, że jeśli proste p i CD nie są rozłączne, to q i AB są współpłaszczyznowe.
  31. Iteracje i dzielenie wielomianów
    Problem rozwiązany przez użytkownika przemk20
  32. Równanie z funkcjami
    Problem rozwiązany przez użytkownika Wasilewski
  33. Liczba Wiliamsa
    Problem rozwiązany przez użytkownika Wasilewski
  34. Własność z najmniejszą wielokrotnością
    Problem rozwiązany przez użytkownika mol_ksiazkowy
  35. Punkty w kwadracie
    Problem rozwiązany przez użytkownika robin5hood
  36. Kwadrat
    Problem rozwiązany przez użytkownika xiikzodz
  37. Dwie funkcje
    Problem rozwiązany przez użytkownika Sir George
  38. Suma potęg
    Problem rozwiązany przez użytkowników Sylwek i mol_ksiazkowy
  39. Suma szeregu
    Problem rozwiązany przez użytkowników przemk20 i Sir George
  40. Wykazać, że funkcja ma co najmniej n miejsc zerowych
    Problem rozwiązany przez użytkownika Wasilewski
  41. Oblicz granicę
    Problem rozwiązany przez użytkownika luka52
  42. Układ równań
    Problem rozwiązany przez użytkownika mol_ksiazkowy
  43. W czworokącie wpukłym ABCD przekątne AC i BD
    Problem rozwiązany przez użytkownika Sylwek
  44. Granica ciągu
    Problem rozwiązany przez użytkownika robin5hood
  45. Nierówność w przestrzeni
    Problem rozwiązany przez użytkownika mol_ksiazkowy
  46. Dwusieczna a trójkąt prostokątny
    Problem rozwiązany przez użytkownika mol_ksiazkowy
  47. Kresy sumy długości boków
    Wyznacz oba kresy, tj. górny (\(\displaystyle{ M}\)) i dolny (\(\displaystyle{ m}\)) wyrażenia \(\displaystyle{ W}\), będącego sumą długości boków i przekątnych czworokąta wypukłego na płaszczyźnie i o polu jednostkowym. Kiedy (o ile są) są one realizowane. Podaj wszystkie stosowne rachunki i (lub) ewentualne uogólnienie.
  48. Funkcja okresowa
    Problem rozwiązany przez użytkownika andkom
  49. Styczne - dowód
    Problem rozwiązany przez użytkownika binaj
  50. Czworokąt- wykazanie nierówności
    Problem rozwiązany przez użytkownika mp2
  51. Trójkąt, punkt określony równością
    Problem rozwiązany przez użytkowników Wasilewski i Sylwek
  52. Granica z symbolem Newtona
    Problem rozwiązany przez użytkownika Wasilewski
  53. Ciąg o wahaniu skończonym
    Problem rozwiązany przez użytkownika Wasilewski
  54. Graf o 17 wierzchołkach
    Problem rozwiązany przez użytkownika Sylwek
  55. Sześciokąt i koła
    Problem rozwiązany przez użytkowników limes123 i Sylwek
  56. Trzy zadania
    1.Środki krawędzi czworościanu leżą na jednej sferze. Wyznacz maksimum objętości tego czworościanu.

    3.Niech ABC będzie trójkątem ostrokątnym, zaś M, N i P - rzutami prostokątnymi środka ciężkości tego trójkąta na boki BC, CA, AB. Udowodnij, że \(\displaystyle{ \frac{4}{27} < \frac{S(MNP)}{S(ABC)} \leqslant \frac{1}{4}}\) Gdzie \(\displaystyle{ S(XYZ)}\) oznacza oczywiście pole trójkąta XYZ.
  57. Zbiór części ułamkowych gęsty w (0, 1)
    Problem rozwiązany przez użytkownika ZetaX
  58. Równość w trójkącie
    W trójkącie \(\displaystyle{ ABC, K, L, M}\), są spodkami wysokości odpowiednio na przeciwko wierzchołków \(\displaystyle{ A, B, C}\), wykaż, że;

    \(\displaystyle{ \frac{|AM||BK|}{|AK|}+\frac{|CL||BK|}{|BL|}+\frac{|AM||CL|}{|CM|}=h_1+h_2+h_3-2(R+r)}\)

    gdzie \(\displaystyle{ R, r}\) są odpowiednio promieniami okręgów opisanego i wpisanego
  59. Wykazać tożsamość
    Problem rozwiązany przez użytkownika ironleaf
  60. Równanie wektorowe dla trójkąta
    Problem rozwiązany przez użytkownika Sylwek
  61. Równanie w zbiorze liczb naturalnych
    Rozwiąż równanie w zbiorze liczb naturalnych
    \(\displaystyle{ a^x+\left(2a+1\right)^y=\left(a+1\right)^z}\) dla danego \(\displaystyle{ a\in \NN - \{1\}}\) i \(\displaystyle{ x,y,z\in \NN\cup\{0\}}\)
  62. Twierdzenie Gegenbauera
    Problem rozwiązany przez użytkownika Maciej87
  63. Ciekawa kombinatoryka
    1. Dwudziestu pięciu hobbitów podzieliło działkę 5x5 na działki 1x1. Zaden hobbit nie chce miec wroga za sasiada (ani nawet zeby jego dzialka stykala sie wierzcholkami z sasiadem). Wiadomo ze zaden nie poklocil sie z wiecej niz trzema innymi hobbitami. Udowodnic, ze mozna ich tak rozstawic, zeby kazdy byl otoczony tylko przyjaciolmi. (wsk. -> udowodnic, ze jedli sasiaduja ze soba jacys wrogowie to mozna zminiejszyc liczbe takich sasiadow).
  64. Miejsce geometryczne
    Miejsce geometryczne zbioru punktów \(\displaystyle{ X}\) z wnętrza trójkąta ostrokątnego \(\displaystyle{ ABC}\) określane jest równością: \(\displaystyle{ AB\cdot BC\cdot CA
    \ =\
    XA\cdot AB\cdot BX+XB\cdot BC\cdot CX+XC\cdot CA \cdot AX}\)


    Wyznaczyć X
  65. Nierówność podłogi
    Problem rozwiązany przez użytkownika Negative_3
  66. Ustawienie klocków
    Mamy \(\displaystyle{ \frac{n(n+1)}{2}}\) klocków z liczbami: jeden klocek z "1", dwa klocki z "2", ..., n klocków z "\(\displaystyle{ n}\)".

    Dla jakich \(\displaystyle{ n}\) można te klocki tak ustawić by
    między każdymi dwoma \(\displaystyle{ n}\)-ami był jeden klocek,
    między kazdymi dwoma klockami z \(\displaystyle{ n-1}\) dwa inne klocki, itd.
    wreszcie między klockami z dwójkami \(\displaystyle{ n-1}\) klocków ??




    ----------------------------------------------------------------------------
    Problemy, które pomimo długiego oczekiwania w pierwszym poście tego tematu, nie doczekały się pełnego rozwiązania:
    ----------------------------------------------------------------------------
  67. Numerowanie
    Problem połowicznie rozwiązany przez użytkownika Sylwek
    Mamy 2007-kąt foremny. Na każdym wierzchołku jest punkt i na każdym środku boku też jest punkt, (czyli w sumie 4014 punktów). Ponumerować te punkty liczbami naturalnymi od 1 do 4014 tak, żeby na każdym boku (tzn. wierzchołek, środek boku, drugi wierzchołek) suma liczb była wszędzie taka sama. Czy takie ponumerowanie jest możliwe dla każdego n-kąta foremnego, czy tylko dla n = 2007?
  68. Trzy zadania
    2. Znajdź wszystkie całkowite \(\displaystyle{ x,y}\), takie że liczba \(\displaystyle{ \frac{x^2+y^2}{xy-1}}\) również jest liczbą całkowitą.
  69. Szesciany
    Problem rozwiązany przez użytkownika Sylwek
  70. Niestandardowa granica
    Udowodnij, że:
    \(\displaystyle{ \lim_{n\to \infty} \{\{3^{n}\}\}=+\infty}\), gdzie:
    \(\displaystyle{ \{\{x\}\}}\) to suma cyfr liczby x.
  71. Istnienie nieskończonego zbioru
    Czy istnieje nieskończony zbiór liczb naturalnych, taki, że suma elementów każdego jego skończonego podzbioru jest potęgą liczby naturalnej?
  72. Suma potęg 10
    Czy suma dwóch lub więcej potęg liczby 10 o wykładnikach naturalnych które się nie powtarzają może być parzystą potęgą liczby naturalnej?
  73. Odległości punktu od boków - wykaż
    Problem rozwiązany przez użytkownika Wasilewski
  74. 2 równania funkcyjne
    2)Znajdź funkcje \(\displaystyle{ f: \mathbb{R}\rightarrow \mathbb{R}}\) takie, że:
    \(\displaystyle{ f((3 + \sqrt {2})x) + f((1 + 3\sqrt {2})x) = 2f((2 + 2\sqrt {2})x)}\)
  75. Nierówność z funkcją Euler'a
    Udowodnij, że dla dowolnych \(\displaystyle{ a,b \in N}\) takich, że \(\displaystyle{ a<b}\) istnieje takie \(\displaystyle{ n\in N}\), że spełniona jest nierówność \(\displaystyle{ a<\frac{\varphi(n+2)}{\varphi(n)}<b}\)
  76. Równanie z parametrem
    Znaleźć wszystkie \(\displaystyle{ c>0}\) dla których istnieją dokładnie 3 pary \(\displaystyle{ (x,y)}\) liczb naturalnych takie ze \(\displaystyle{ xy^2 - y^2 - x + y=c}\)
  77. Zestaw zadań z KMDO
    5. Problem rozwiązany przez użytkownika Sylwek
  78. Trzy lematy
    Problem połowicznie rozwiązany przez użytkownika mol_ksiazkowy
    c) Dowiesc ze jesli F jest polem wielokata wypuklego , a L dlugoscia lamanej go ograniczajacej, zas R promien najmniejszego koła zawierajacego ten wielokat, to
    \(\displaystyle{ \pi R^2-LR+F \le 0}\) i... Kiedy zachodzi rownosc??
  79. 2x(nierówność geometryczna+funkcja)
    1. W trójkącie ABC punkt M jest środkiem ciężkości. Okrąg opisany na trójkącie AMC jest styczny do prostej AB. Wykaż, że
    \(\displaystyle{ \sin CAM+ \sin CBM \leqslant \frac{2}{ \sqrt{3} }}\)

    3. Udowodnij, że jeżeli M jest dowolnym punktem wewnętrznym trójkąta ABC, to:
    \(\displaystyle{ \min(MA,MB,MC)+MA+MB+MC<AB+BC+CA}\)




    ----------------------------------------------------------------------------
    Tematy zawierające kilkanaście zadań:
    ----------------------------------------------------------------------------
  80. :arrow: Obóz matematyczny Gryfino 2005
  81. :arrow: Obóz matematyczny Gryfino 2006
  82. :arrow: Świętokrzyskie warsztaty, wrzesień 2007
  83. :arrow: Mix matematyczny (9) - geometria
  84. :arrow: Mix matematyczny (14) - geometria
  85. :arrow: Maxi-mix (TEORIA LICZB)- dualne do Mathlinks
  86. :arrow: Mix matematyczny (15) - kombinatoryka
  87. :arrow: Mix matematyczny (16) - teoria liczb
  88. :arrow: Delta i Kwant - zestawienie kącików zadaniowych
  89. :arrow: Świętokrzyskie warsztaty, wrzesień 2008


Powodzenia!
frej

[MIX] Nierozwiązane problemy

Post autor: frej »

Oczywiście, jak większość użytkowników się orientuje, temat ten jest dziełem Sylwka. Ponieważ jego aktywność prawdopodobnie zmaleje w czasie przedłużonych wakacji ( maturzysta ), do odwołania tematem będę zajmował się ja. Nic w działaniu tego tematu nie ulegnie zmianie, postaram się nie zepsuć tego, co Sylwek wraz z molem_książkowym stworzyli. Dwa poprzednie posty zostały stworzone bez mojej pracy i są bez wątpliwości dziełem Sylwka ( być może jeszcze kogoś, kto mu w tym pomagał ). Mam nadzieję, że uda się rozwiązać trwające nadal problemy. O zadaniach ciekawych i dość wymagających, które należałoby tu umieścić proszę informować mnie na

Powodzenia w rozwiązywaniu zadań
frej

[MIX] Nierozwiązane problemy

Post autor: frej »

Niestety, a może na szczęście, forum się rozrosło i liczba ciekawych nierozwiązanych problemów znacząco wzrosła. Z tego powodu dalsze prowadzenie, czyli modyfikowanie tego tematu, jest dość pracochłonne itp. Wyżej zarysowane powody sprawiają, że temat ten zostanie zamknięty. Nie wykluczam, że kiedyś temat ten zostanie zaktualizowany, lecz na pewno nie teraz.

Pozdrawiam
Awatar użytkownika
mol_ksiazkowy
Użytkownik
Użytkownik
Posty: 11263
Rejestracja: 9 maja 2006, o 12:35
Płeć: Mężczyzna
Lokalizacja: Kraków
Podziękował: 3140 razy
Pomógł: 747 razy

[MIX] Nierozwiązane problemy

Post autor: mol_ksiazkowy »

Niestety, a może na szczęście, forum się rozrosło i liczba ciekawych nierozwiązanych problemów znacząco wzrosła. Z tego powodu dalsze prowadzenie, czyli modyfikowanie tego tematu, jest dość pracochłonne itp. Wyżej zarysowane powody sprawiają, że temat ten zostanie zamknięty. Nie wykluczam, że kiedyś temat ten zostanie zaktualizowany, lecz na pewno nie teraz.
Nierozwiązane problemy III

Niektóre zadania są z tzw. Mix-ów; a inne nie...
Problemy te nie są rozwiązane
(lub tylko w sposób częściowy)…
Wszelkie informacje, jakieś błędy ,itd. do
mol_ksiazkowy lub Ponewor
Jest \(\displaystyle{ p=101}\) zadań:

Są one i z takiej tematyki:
algebra
geometria
Nierówności;
kombinatoryka
probablistyka, grafy itd. inne.
A więc:

1. Lemat Spernera
Trójkąt T został podzielony na trójkąty, tak że każde dwa trójkąty są albo rozłączne, albo mają wspólny bok lub wierzchołek. wierzcholki trojkątów kolorujemy liczbami 0,1,2. tak ze dla kazdego koloru istnieje bok trójkąta T taki ze zaden wierzcholek lezacy na tym boku nie jest nim pokolorowany (w szczególności wierzchołki T są różnokolorowe). Udowodnij ze liczba trojkolorowych trójkątów jest nieparzysta.
Ukryta treść:    
Dumel
[Kombinatoryka] Lemat Spernera

2. Reszty modulo
Niech \(\displaystyle{ a,b \in \mathbb{N}}\) będą takie, że \(\displaystyle{ \forall_{p \in \mathbb{P}} \ a \leq b (mod \ p)}\), gdzie \(\displaystyle{ \mathbb{P}}\) to zbiór liczb pierwszych.
Pokazać, że \(\displaystyle{ a=b}\)
Piotr Rutkowski
[MIX] UW kontratakuje (Mix zadań)

3. Istniejące permutacje
Przypuśćmy, że \(\displaystyle{ a _{1}, a _{2},...,a _{n}}\) są liczbami całkowitymi takimi że \(\displaystyle{ n| a _{1}+a _{2}+...+a _{n}}\). Pokaż, że istnieją dwie permutacje \(\displaystyle{ \left( b _{1}, b _{2},..., b _{n} \right)}\) i \(\displaystyle{ \left( c _{1}, c _{2},..., c _{n} \right)}\) zbioru \(\displaystyle{ \left( 1,2,...,n\right)}\) takie, że dla każdego \(\displaystyle{ 1\le i \le n}\)
\(\displaystyle{ n| a _{i}-b _{i}-c _{i}}\).
Burii
[Kombinatoryka][Teoria liczb] Dwie permutacje i podzielność.

4. Pokrycie kwadratu
Tożsamość \(\displaystyle{ 1^2+2^2+3^2+...+24^2=70^2}\)
nasuwa przypuszczenie, że możliwe jest pokrycie kwadratu \(\displaystyle{ 70 \times70}\) 24-ma kwadratami o wymiarach \(\displaystyle{ 1 \times 1}\), \(\displaystyle{ 2 \times 2}\), \(\displaystyle{ 3 \times 3}\), ....., \(\displaystyle{ 24 \times 24}\) Czy takie pokrycie istotnie jest możliwe?
mol_ksiazkowy
[MIX] max mix

5. Zbiór punktów
Dane są punkt \(\displaystyle{ M}\), prosta \(\displaystyle{ l}\) oraz wektor \(\displaystyle{ \vec{p}}\) . Symetria względem prostej \(\displaystyle{ l}\) przekształca dowolny punkt \(\displaystyle{ X}\) w \(\displaystyle{ X^{'}}\), zaś przesuniecie \(\displaystyle{ \vec{p}}\) przekształca \(\displaystyle{ X^{'}}\) w \(\displaystyle{ X^{''}}\). Jaki zbiór tworzą punkty \(\displaystyle{ X}\), dla których punkty \(\displaystyle{ M}\), \(\displaystyle{ X}\), \(\displaystyle{ X^{''}}\) są współliniowe?
Brycho
[MIX][Planimetria] Punktem w Dwuwymiarze było niegdyś się... - zestaw zadań

6. Mix, Kolory i liczby
Udowodnić, że jest możliwe "pokolorowanie" każdej liczby wymiernej q dodatniej jednym z dwóch kolorów na czerwono lub biało w taki sposób, aby \(\displaystyle{ q}\) i \(\displaystyle{ \frac{1}{q}}\) były jednakowego koloru, zaś \(\displaystyle{ q}\) i \(\displaystyle{ q+1}\) różnych kolorów.
juzef
[MIX] Obóz matematyczny Gryfino 2006
Problem rozwiązany przez użytkownika Ponewor.
7. Proste równania
Mamy równanie:

\(\displaystyle{ 2^{x+y}-3^{y}=c \cdot p}\)

Wszystkie zmienne to liczby naturalne. Przyjmijmy, że \(\displaystyle{ c=5}\), czy istnieje skończenie wiele par liczb \(\displaystyle{ x}\) i \(\displaystyle{ y}\) spełniających to równanie?

Czy dla dowolnie przyjętego \(\displaystyle{ c}\) (poza \(\displaystyle{ c=1}\)) istnieje skończenie wiele par liczb \(\displaystyle{ x}\) i \(\displaystyle{ y}\) spełniających to równanie?
matemix
Równanie diofantyczne 3 zmiennych

8. Mix, IMO 5, Szacowanie ciągu
Dany jest ciąg (\(\displaystyle{ a_n}\)) spełniający warunki:
\(\displaystyle{ a_0 = 4}\)
\(\displaystyle{ a_{n+1} = a_n^2 - 2}\).
Udowodnić, że \(\displaystyle{ a_{n+1} > 2 \sqrt{3} a_n...a_1a_0}\).
jerzozwierz
[MIX] Obóz przygotowujący do IMO
Problem rozwiązany przez użytkownika mol_ksiazkowy.
9. Mix, Liczby naturalne, liczby pierwsze
Dane są liczby całkowite \(\displaystyle{ m, n}\) spełniające \(\displaystyle{ 0<n<m}\). Udowodnić, że jeśli liczby \(\displaystyle{ a^m - 1}\) i \(\displaystyle{ a^n - 1}\) mają te same dzielniki pierwsze, to \(\displaystyle{ a+1}\) jest potęgą dwójki.
jerzozwierz
[MIX][Teoria liczb] Liczby pierwsze, liczby naturalne
Problem rozwiązany przez użytkownika KPR.
10. Suma szeregu
Wykaż, że:
\(\displaystyle{ \frac{1}{2}\;-\;\frac{1\cdot 3}{2\cdot 4\cdot 6}\;+\;\frac{1\cdot 3\cdot 5\cdot 7}{2\cdot 4\cdot 6\cdot 8\cdot 10}-\;\frac{1\cdot 3\cdot 5\cdot 7\cdot 9\cdot 11}{2\cdot 4\cdot 6\cdot 8\cdot 10\cdot 12\cdot 14}\;+\;...=\;\sqrt{1\;-\;\frac{1}{\sqrt{2}}}}\)
robin5hood
[Analiza][Ciągi] Suma sezeregu
Problem rozwiązany przez użytkownika JakimPL.
11. Funkcja i okres
Niech \(\displaystyle{ a \neq b}\) i \(\displaystyle{ f}\) jest ciągła taka, że \(\displaystyle{ \frac{f(x)}{x^2}}\) dąży do 0 kiedy \(\displaystyle{ x}\) dąży do nieskończoności lub minus nieskończoności. Niech \(\displaystyle{ f(x+a)+f(x+b)=\frac{f(2x)}{2}}\). Uzasadnić że \(\displaystyle{ f}\) jest okresowa.
tatteredspire
[Równania funkcyjne] funkcja okresowa

12. Mix, Jeden kwadrat
Wykaż lub obal: Jeśli \(\displaystyle{ x, y}\) są liczbami naturalnymi takimi, że \(\displaystyle{ xy+ x}\) oraz \(\displaystyle{ xy+y}\) są kwadratami pewnych liczb całkowitych, to dokładnie jedna z liczb \(\displaystyle{ x, y}\) jest kwadratem liczby naturalnej.
Uwaga: Takimi są np. \(\displaystyle{ x=4 \ y=80}\)
Ukryta treść:    
mol_ksiazkowy
[MIX] Zestaw zadań na sezon ogórkowy II

13. Punkty na kuli
Na powierzchni kuli wybieramy losowo 4 punkty. Ile wynosi prawdopodobieństwo, ze 4 z nich leżą na powierzchni pewnej połkuli ?
xiikzodz
[Prawdopodobieństwo] Punkty na kuli

14. Dwa kolory i trójkąt
Każdy punkt płaszczyzny malujemy na jeden z dwóch kolorów. Pokazać, że istnieje trójkąt którego wierzchołki i środek okręgu wpisanego są jednokolorowe.
burii
[Kombinatoryka] Dwa kolory i trójkąt

15. Mix, Operacje w tablicy
W każdym wierszu i w każdej kolumnie kwadratowej tablicy \(\displaystyle{ n \times n}\) stoi dokładnie jeden raz liczba \(\displaystyle{ 1}\), dokładnie jeden raz liczba \(\displaystyle{ -1}\) i \(\displaystyle{ n-2}\) razy liczba \(\displaystyle{ 0}\). Operacja elementarna polega na zamianie miejscami dwóch kolumn lub zamianie miejscami dwóch wierszy. Dowieść, że za pomocą pewnej ilości operacji elementarnych można zamienić miejscami położenia \(\displaystyle{ +1}\) i \(\displaystyle{ -1}\).
Arbooz
viewtopic.php?t=4031

16. Łatwa teoria liczb
Witam, zadanie jest z okręgowych LX OM:

Dane są takie liczby całkowite \(\displaystyle{ a}\) i \(\displaystyle{ b}\), że \(\displaystyle{ a > b >1}\) oraz liczba \(\displaystyle{ ab +1}\) jest podzielna przez \(\displaystyle{ a+ b}\), zaś liczba \(\displaystyle{ ab - 1}\) jest podzielna przez \(\displaystyle{ a-b}\). Wykazać, że \(\displaystyle{ a < b\sqrt{3}}\)

Czy jest ktoś w stanie zaprezentować rozwiązanie zasadniczo różniące się od wzorcowego? Bo nie wierzę, że istnieje tylko jedna droga prowadząca do celu.
Ukryta treść:    
Marcinek665
[Teoria liczb] Łatwa teoria liczb z okręgowych.
Problem rozwiązany przez użytkownika Ponewor.
17. Mix 31, Trójkąty i węzły
Jakie trójkąty prostokątne można narysować na płaszczyźnie z siatką kwadratową tak, że przeciwprostokątna leży na jednej z linii siatki, a wszystkie wierzchołki leżą w węzłach (tj. punktach kratowych) tej siatki ?
mol_ksiazkowy
[MIX] Mix matematyczny (31)

18. Suma potęg z podzielnością
Czy warunek \(\displaystyle{ n \mid \sum_{i=1}^{n-1} i^{n-1}+1}\) może spełniać \(\displaystyle{ k}\) kolejnych (niekoniecznie \(\displaystyle{ k}\) początkowych) liczb naturalnych, gdzie \(\displaystyle{ k}\) jest większe od dowolnej liczby naturalnej \(\displaystyle{ m}\)?
tatteredspire
Suma potęg z podzielnością
Problem rozwiązany przez użytkownika Marcin7Cd.
19. Mix, zadanie 27
Udowodnić nierówność \(\displaystyle{ \frac{A+a+B+b}{A+a+B+b+c+r} + \frac{B+b+C+c}{B+b+C+c+a+r} > \frac{C+c+A+a}{C+c+A+a+b+r}}\)
w której wszystkie litery oznaczają liczby dodatnie
Ukryta treść:    
mol_ksiazkowy
[MIX] Zestaw zadań na sezon ogórkowy
Problem rozwiązany przez użytkownika Ponewor.
20. Tablica \(\displaystyle{ 9 \times 9}\)
W kwadratowa tablice składającą się z \(\displaystyle{ 81}\) jednakowych kwadratowych pól wpisano wszystkie liczby naturalne od \(\displaystyle{ 1}\) do \(\displaystyle{ 81}\). Udowodnij, ze dla dowolnego ułożenia liczb istnieją dwie sąsiednie różniące się co najmniej o \(\displaystyle{ 6}\). Przez sąsiednie rozumiemy liczby wpisane w pola o wspólnej krawędzi.
przemson
[Kombinatoryka] W kwadratową tablicę wpisano liczby ...
Problem rozwiązany przez użytkownika Ponewor.
21. Reszta
Dla jakich liczb pierwszych nieparzystych liczba \(\displaystyle{ -3}\) jest resztą kwadratową modulo \(\displaystyle{ p}\) ?
Ukryta treść:    
fala21
Reszty kwadratowe
Problem rozwiązany przez użytkownika Ponewor.
22. Ciekawa geo; Czworokąt
Niech \(\displaystyle{ O}\) będzie punktem, w którym przecinają się przekątne czworokąta wypukłego \(\displaystyle{ ABCD}\). Okręgi opisane na trójkątach \(\displaystyle{ OAD}\) i \(\displaystyle{ OBC}\) i przecinają się w punktach \(\displaystyle{ O}\) i \(\displaystyle{ M}\) . Prosta \(\displaystyle{ OM}\) przecina okręgi opisane na trójkątach \(\displaystyle{ OAB}\) i \(\displaystyle{ OCD}\) w punktach \(\displaystyle{ P}\) i \(\displaystyle{ Q}\). Pokaż, że \(\displaystyle{ M}\) jest środkiem \(\displaystyle{ PQ}\).
tometomek91
[Planimetria] Ciekawa geo: pokazać, że M jest srodkiem.

23. Konstrukcja
Dane są trzy okręgi, w tym dwa styczne. Skonstruować okrąg styczny do tych trzech okręgów
porfirion
[Planimetria] Trzy zadanka z inwersji

24. Problem przedstawialności
Udowodnij, że istnieje tylko skończenie wiele liczb naturalnych \(\displaystyle{ n}\), których nie da się przedstawić w postaci sumy kwadratów różnych liczb naturalnych.
ElEski
[Teoria liczb] Przyjemny problem przedstawialności
Problem rozwiązany przez użytkownika ElEski.

25. Wykazanie nierówności
Niech \(\displaystyle{ x, y> 0}\) oraz \(\displaystyle{ 3(x+y) \ge 2(xy+1)}\).Wykaż \(\displaystyle{ 9(x^{3}+y^{3}) \ge x^{3}y^{3}+1}\)
marek12
[Nierówności] wykazanie nierówności

26. Bijekcja jako suma bijekcji
Wykaż że każda bijekcja \(\displaystyle{ f : Z \mapsto Z}\) jest sumą dwóch funkcji \(\displaystyle{ g, h : Z \mapsto Z}\) które są bijekcjami.
Burii
[Funkcje] Bijekcja jako suma bijekcji.
Problem rozwiązany przez użytkownika Zordon.
27. Nierówność Newberga Pedoe’a
Gdy dane są dwa trójkąty \(\displaystyle{ t1}\) oraz \(\displaystyle{ t2}\) o bokach
\(\displaystyle{ a_1, b_ 1, c_ 1}\) oraz \(\displaystyle{ a_2, b_ 2, c_ 2}\)
i o polach \(\displaystyle{ \Delta_1 , \ \Delta_2}\) to :
\(\displaystyle{ a_1^2(b_2^2 +c_2^2-a_2^2) + b_1^2(c_2^2 +a_2^2-b_2^2)+ c_1^2(a_2^2 +b_2^2-c_2^2) \geq 16 \Delta_1 \Delta_2}\)
i kiedy zachodzi równość ?
mol_ksiazkowy
Nierówność Newberga Pedoe'a
Problem rozwiązany przez użytkownika mol_ksiazkowy.
28. Własność czworościanu
Udowodnić, że jeżeli kula wpisana w czworościan ma wspólny środek z kulą opisaną na tym czworościanie, to sumy kątów płaskich przy każdym wierzchołku tego czworościanu są równe \(\displaystyle{ 180^{ \circ}}\)
patry93
[Stereometria] Kule i czworościan, kąty płaskie

29. Znalezienie minimum
Niech \(\displaystyle{ a, b \in N}\) i \(\displaystyle{ a \neq 1,b \neq 1}\), gdzie \(\displaystyle{ a^{9a}=b^{2b}}\). Znaleźć minimum \(\displaystyle{ 2a+b}\).
darek20
[Nierówności] Znalezienie minimum
Problem rozwiązany przez użytkownika mol_ksiazkowy.

30. Układ wektorów
Danych jest \(\displaystyle{ n}\) wektorów \(\displaystyle{ \vec{v_j}}\) na płaszczyźnie i wszystkie one są zaczepione w punkcie \(\displaystyle{ O(0,0)}\) oraz \(\displaystyle{ |\vec{v_j}| =1}\) dla \(\displaystyle{ j=1,..., n}\). Wykaż że jeśli dla pewnego \(\displaystyle{ k}\), takiego że \(\displaystyle{ 2k <n}\) nie mniej niż \(\displaystyle{ k}\) wektorów leży po obu stronach dowolnej prostej do której należy punkt \(\displaystyle{ O}\), to \(\displaystyle{ |v| \leq n-2k}\), gdzie \(\displaystyle{ v}\) jest sumą wektorów \(\displaystyle{ \vec{v_j}}\). Czy może zachodzić równość ?
mol_ksiazkowy
[Planimetria] Układ wektorów

31. Zbiór skończony
Niech \(\displaystyle{ M}\) będzie zbiorem skończonym, który zawiera co najmniej 2 różne dodatnie liczby rzeczywiste. Załóżmy że dla dowolnego \(\displaystyle{ a \in M}\), istnieją \(\displaystyle{ b, c \in M}\) (\(\displaystyle{ a, b, c}\) niekoniecznie różne) takie, że \(\displaystyle{ a = 1 +\frac{b}{c}}\). Udowodnić, że można znaleźć \(\displaystyle{ x, y \in M \ (x \neq y)}\) spełniające warunek \(\displaystyle{ x + y> 4.}\)
darek20
zbiór skończony

32. Równanie diofantyczne; HARDCORE
Rozwiązać w \(\displaystyle{ Z+}\): \(\displaystyle{ (a^{2}+b^{2}-c^{2})(a+b)=2ab^{2}}\)
Andix
równanie diofantyczne (niezły hardcore)

33. Mix, Zawodnicy i zadania
Pewne zawody matematyczne odbyły się w dwóch dniach. Rozwiązywano łącznie 28 zadań. Dla dowolnej pary dwóch zadań znalazło się dokładnie dwóch zawodników, którzy je rozwiązali. Każdy zawodnik rozwiązał dokładnie 7 zadań. Wykazać, że był taki zawodnik, który w pierwszy dzień albo nie rozwiązał żadnego zadania albo rozwiązał co najmniej cztery
Ukryta treść:    
mol_ksiazkowy
[MIX] Mix matematyczny (28)

34. Mix 27, Rozkład
Dane są trzy liczby naturalne \(\displaystyle{ x, y, z}\) powiązane zaleznościa \(\displaystyle{ xy= z^2+1}\). Wykazać, że istnieją liczby całkowite \(\displaystyle{ a, b, c, d}\) takie że
\(\displaystyle{ \begin{cases} x=a^2+b^2\\y=c^2+d^2\\z=ac+bd\end{cases}}\)
Dać przykład (podając \(\displaystyle{ a,b,c,d}\), dla \(\displaystyle{ x=13, \ y=5, \ z=8}\))
mol_ksiazkowy
[MIX] Mix matematyczny (27)
Problem rozwiązany przez użytkownika mol_ksiazkowy .
35. Równanie Bacheta
Równanie diofantyczne:
\(\displaystyle{ x^{3} = y^{2} + k}\)
zależne od parametru całkowitego \(\displaystyle{ k}\) nazywa się równaniem Bacheta (albo równaniem Mordella).
Pokaż, że istnieje nieskończenie wiele parametrów \(\displaystyle{ k}\) takich, że równanie to nie ma rozwiązania w liczbach całkowitych.
max
[Teoria liczb] Nieskończenie wiele równań Bacheta bez rozwiązań
Problem rozwiązany przez użytkownika Ponewor.
36. Ciąg i różnice
Czy istnieje ściśle rosnący ciąg liczb całkowitych dodatnich \(\displaystyle{ a_n}\) taki, że \(\displaystyle{ a_n \leq n^3}\) oraz każda liczba całkowita dodatnia występuje dokładnie raz jako różnica dwóch elementów tego ciągu?
Kolega Damiana
[Ciągi] Ciąg i różnice

37. Lemat o liczbach zespolonych
Dany jest skończony ciąg liczb zespolonych: \(\displaystyle{ z_1, ....,z_n}\), wykaż ze można wybrać \(\displaystyle{ S \subset \{ 1, ...,n\}}\) t. że zachodzi (*). Dać też przykład, że uogólnienie na przypadek nieskończony nie działa (a może jednak....?!). Wszelkie metody i uwagi mile widziane. etc
(*) \(\displaystyle{ |\sum_{j \in S} z_j| \geq \frac{1}{6} \sum_{j=1}^n |z_j|}\)
mol_ksiazkowy
sprytny lemat
Problem rozwiązany przez użytkownika mol_ksiazkowy.
38. Ciąg stopni
Czy każde dwa grafy SPÓJNE o takim samym ciągu stopni są izomorficzne?
Proszę o pomoc w postaci odpowiedzi z uzasadnieniem, bądź kontrprzykładu
MatizMac
grafy izomorficzne
Problem rozwiązany przez użytkownika mol_ksiazkowy.
39. Kombi z wartością bezwzględną
Wśród liczb \(\displaystyle{ 1,2,3,...,n+1}\) jedna liczba została skreślona, zaś pozostałe są ustawione w kolejności \(\displaystyle{ a_{1} ,a _{2} ,..., a_{n}}\) w taki sposób , aby wszystkie \(\displaystyle{ n}\) wartości bezwzględnych \(\displaystyle{ \left| a _{1}-a _{2} \right| ,\left| a _{2}- a_{3} \right| ,..,\left|a _{n} -a _{1} \right|}\) różniły się pomiędzy sobą. Dla jakich liczb naturalnych\(\displaystyle{ n \ge 3}\) można to zrobić ?
cyberciq
[Kombinatoryka] Kombi z wartością bezwzględną
Problem rozwiązany przez użytkownika mol_ksiazkowy.
40. Nierówność w trójkącie
Pokaż że jeśli \(\displaystyle{ a,b,c}\) są bokami trójkata oraz \(\displaystyle{ c\geq a}\) to \(\displaystyle{ \frac{b^{2}+c^{2}}{a^{2}}\geq \frac{2\sqrt{3}c\cdot\sin\beta -a}{b+c}}\)
gdzie \(\displaystyle{ \beta}\) jest kątem \(\displaystyle{ ABC}\)
rochaj
[Nierówności][Planimetria] Nierówność z bokami trójkata
Problem rozwiązany przez użytkownika mol_ksiazkowy.
41. Granica ciągu rekurencyjnego
Niech \(\displaystyle{ \{a_n\}_{n = 0}^{\infty}}\) : \(\displaystyle{ a_0 = a_1 = 1, a_{n + 2} = \frac {1}{a_{n + 1}} + \frac {1}{a_n}}\) .Oblicz \(\displaystyle{ \lim_{n \to \infty}a_n}\)
Ukryta treść:    
robin5hood
granica ciagu rekurencyjnego

42. Mix, Trójka Cevy
Trzy punkty leżące na trzech bokach trójkąta nazwiemy trójką Cevy, jeżeli proste łączące te punkty z wierzchołkami przecinają się w jednym punkcie. Przypuśćmy, że wykreśliliśmy okrąg przez punkty tworzące trójkę Cevy otrzymując trzy nowe punkty na bokach trójkąta. Udowodnij, że nowe punkt także tworzą trójkę Cevy.
Sylwek
[MIX] Świętokrzyskie warsztaty, wrzesień 2007
Problem rozwiązany przez użytkownika Ponewor.
43. Wykazanie nierówności
Niech \(\displaystyle{ a,b,c, d}\) całkowite i takie ze \(\displaystyle{ \lvert\ ad-bc \rvert=1}\) oraz \(\displaystyle{ \lvert\ a \rvert> \lvert\ c \rvert}\) . Wykaż że \(\displaystyle{ a^{2}+ab+b^{2}\ge c^{2}+cd+d^{2}}\).
darek20
[Nierówności] wykazanie nierównosci w liczbach całkowitych

44. Podział zbioru
dla jakich \(\displaystyle{ n}\) można zbiór \(\displaystyle{ \{1,2,...,3n\}}\) podzielić na \(\displaystyle{ n}\) trójelementowych podzbiorów takich że w każdym z nich największa liczba jest równa sumie dwóch mniejszych?
moje wypociny:    
Dumel
[Kombinatoryka] podział zbioru na 3-el. podzbiory

45. Nierówność z maksimum
Dane są liczby rzeczywiste \(\displaystyle{ A,B,C>0}\) wykaż nierówność \(\displaystyle{ max(A^2-B, B^2-C,C^2-A) \ge max(A^2-A, B^2-B,C^2-C)}\)
marek12
[MIX] Różne "na wakacje od dziś"
Problem rozwiązany przez użytkownika timon92.
46. Dwa równania
1.Dla \(\displaystyle{ a,b,c \in N}\)
Znajdź wszystkie rozwiązania równania \(\displaystyle{ a^2 + b^3 = c^4}\)
2.Dla \(\displaystyle{ a,b,c,d \in N}\)
Znajdź rozwiązania równania \(\displaystyle{ a^2 + b^3 + c^4 = d^5}\)
Milczek
[Równania][Teoria liczb] Znajdź wszystkie rozwązania
Problem rozwiązany połowicznie przez użytkownika Jakub Gurak.
47. Trójkąty w wielokącie
Mamy \(\displaystyle{ n}\) - kąt wypukły (\(\displaystyle{ n>3}\)) w którym żadne trzy przekątne nie mają wspólnego punktu. Ile jest wszystkich trójkątów powstałych w wyniku podziału tego \(\displaystyle{ n}\)-kąta tymi \(\displaystyle{ \frac{n(n-3)}{2}}\) przekątnymi
zaklopotany93
[Geometria][Kombinatoryka] liczba trójkątów

48. Mix, Ciąg i minimum
Dla ustalonej liczby naturalnej \(\displaystyle{ n \ge 2}\) określamy:
\(\displaystyle{ x_1=n, \ \ y_1=1, \ \ x_{i+1}= \left[ \frac{1}{2}(x_i+y_i) \right], \ \ y_{i+1}= \left[ \frac{n}{x_{i+1}} \right]}\).
Dowieść, że \(\displaystyle{ \text{min}\{x_1, x_2,...,x_n\}=\left[\sqrt{n} \right]}\)
ares41
[MIX] Kilka zadań na deszczowe wieczory
Problem rozwiązany przez użytkownika mol_ksiazkowy.
49. Nierówność z mantysą
Wykaż dla każdej liczby naturalnej bez zera: \(\displaystyle{ \left\lfloor n\sqrt 3\right\rfloor\left\{n\sqrt 3\right\}>\frac{3}{5}}\)
darek20
wykazanie nierówności
Problem rozwiązany przez użytkownika Ponewor.
50. Teoria liczb, zestaw mola, problem 100
Niech \(\displaystyle{ S}\) będzie zbiorem 43 liczb naturalnych nie większych niż \(\displaystyle{ 100}\). Dla każdego podzbioru \(\displaystyle{ X \subset S}\) niech \(\displaystyle{ t_X}\) to będzie iloczyn wszystkich liczb ze zbioru \(\displaystyle{ X}\). Wykaż, ze istnieją dwa rozłączne podzbiory \(\displaystyle{ A}\), \(\displaystyle{ B}\) zbioru \(\displaystyle{ S}\) takie, że \(\displaystyle{ t_At_B^2}\) jest sześcianem liczby naturalnej
mol_ksiazkowy
[Teoria liczb] zestaw mola

51. Własność \(\displaystyle{ k}\) podziału
Dany jest ciąg liczb rzeczywistych \(\displaystyle{ a_1,a_2,...,a_n}\) oraz liczba rzeczywista \(\displaystyle{ M}\). Powiemy, że ten ciąg ma własność \(\displaystyle{ k}\)-podziału (\(\displaystyle{ k\in \mathbb{N}_{>0}}\)) jeżeli istnieje podział tego ciągu na dokładnie k spójnych, niepustych fragmentów, każdy o sumie nieprzekraczającej \(\displaystyle{ M}\).

Udowodnić, że dla dowolnego M jeśli ciąg ma własność \(\displaystyle{ k_1}\)-podziału oraz \(\displaystyle{ k_2}\)-podziału to ma też własność \(\displaystyle{ k}\)-podziału dla wszelkich \(\displaystyle{ k_1\leq k\leq k_2}\).
Zordon
[Kombinatoryka] Własność k-podziału

52. Mix, Dualne do Mathlinks; ostatnie
Niech \(\displaystyle{ A}\) będzie skończonym zbiorem liczb, takim że: dla dowolnego \(\displaystyle{ a \in A}\) istnieją dokładnie dwa elementy \(\displaystyle{ b, c \in A}\) i \(\displaystyle{ b \leq c}\) i \(\displaystyle{ a=b+c}\). Wykaż, ze istnieją parami różne elementy \(\displaystyle{ a_1, ... , a_k \in A}\) takie że: \(\displaystyle{ a_1+...+a_k=0}\)
mol_ksiazkowy
[MIX][Teoria liczb] Maxi-Mix - dualne do Mathlinks

53. Turniej niehamiltonowski
Udowodnij że w każdym niehamiltonowskim turnieju możliwy jest podział wierzchołków na dwie klasy \(\displaystyle{ A}\) i \(\displaystyle{ B}\), taki że dla dowolnych wierzchołków \(\displaystyle{ a \in A}\) i \(\displaystyle{ b \in B}\) krawędź skierowana między nimi ma zwrot \(\displaystyle{ a \to b}\)
Dumel
turniej niehamiltonowski
Problem rozwiązany przez użytkownika kaszubki.
54. Nierówność , dwusieczna
W trójkącie \(\displaystyle{ ABC}\) dwusieczna kąta wychodząca z wierzchołka \(\displaystyle{ A}\) przecina okrąg opisany na tym trójkacie w punkcie \(\displaystyle{ K}\). Jeśli \(\displaystyle{ X}\) jest środkiem \(\displaystyle{ AK}\) pokaż że \(\displaystyle{ BX+CX\geq{AK}}\)
darek20
wykazanie nierówności

55. Mix, Wybór punktów
W prostokącie o polu jednostkowym wybrano pięć punktów takich, że żadne trzy nie są współliniowe. Znajdź najmniejszą liczbę trójkątów o wierzchołkach w trzech z tych pięciu punktów takich, że pole żadnego trójkąta nie przekracza \(\displaystyle{ \frac{1}{4}}\).
snm
[MIX] Mix wakacyjny (24)

56. Losowanie liczb
Ze zbioru \(\displaystyle{ \{1,2,...,n\}}\) wybieramy losowo bez zwracania \(\displaystyle{ k}\) liczb \(\displaystyle{ (k \leqslant n)}\). Oblicz wartość oczekiwaną różnicy między największą a najmniejszą z wylosowanych liczb.
marek12
wartość oczekiwana
Problem rozwiązany przez użytkownika fon_nojman.

57. Kartka z trudnymi, zawody, T9
Czy istnieje funkcja ograniczona i ciągła \(\displaystyle{ f: \mathbb {R} \to \mathbb {R}}\) taka, że dla dowolnych \(\displaystyle{ x,y \in \mathbb {R}}\) :
\(\displaystyle{ (f(x)-f(y))(f(x)+f(y))=f(x-y)f(x+y)}\) ?
Ukryta treść:    
snm
[MIX] IX Warsztaty Matematyczne - kartka z trudnymi
Problem rozwiązany przez użytkownika mol_ksiazkowy.
58. Mix 23, okres
Niech \(\displaystyle{ f: R \mapsto R}\) będzie taka funkcją, iż \(\displaystyle{ |f(x)| \leq 1}\) dla każdego \(\displaystyle{ x \in R}\), oraz zachodzi tożsamość \(\displaystyle{ f(x) + f(x+ \frac{13}{42})= f(x+ \frac{1}{6}) + f(x+ \frac{1}{7})}\). Wykaż, że \(\displaystyle{ f}\) jest funkcją okresową.
mol_ksiazkowy
[MIX] Mix matematyczny (23)
Problem rozwiązany przez użytkownika mol_ksiazkowy.
59. Mix 29, Jeszcze jeden podział
Wskazać (wraz z dowodem ) najmniejszy możliwie \(\displaystyle{ n}\) takie, że przy dowolnym podziale zbioru \(\displaystyle{ S= \{ 1,2,3,...,n \}}\) na dwa podzbiory, jeden z nich (lub być może oba) będzie zawierał elementy \(\displaystyle{ a, b \ a \neq b}\) takie że \(\displaystyle{ ab}\) dzieli się przez \(\displaystyle{ a+b}\)
mol_ksiazkowy
[MIX] Mix Matematyczny 29 (podwójny)

60. Konik i miny
Mamy sobie skoczka który skacze po prostej linii, wykonuje on \(\displaystyle{ n}\) ruchów przy czym każdy jest innej długości i nie większej niż \(\displaystyle{ n}\). Tzn. wykonuje ruchy długości \(\displaystyle{ 1;2;3;....n-1;n}\) przy czym w dowolnej kolejności. Czy jest możliwe, aby ustawiając na drodze \(\displaystyle{ n-1}\) min skoczek nie mógł przejść
exupery
skoczek, linia prosta miny

61. Suma kwadratów i kwadrat sumy
Przekształcić (elementarnie) do postaci sumy kwadratów bądź kwadratu sumy \(\displaystyle{ \sum_{i=1}^{n} {a_i}^n-n \prod_{i=1}^{n} a_i}\) gdzie \(\displaystyle{ a_i \ge 0}\) dla dowolnego \(\displaystyle{ i=1,2,3,...,n}\)
tatteredspire
Przedstawić w postaci sumy kwadratów bądź kwadratu sumy

62. Dobór znaków
Każdy z wektorów (na płaszczyźnie \(\displaystyle{ \mathbb{R}^2}\)) \(\displaystyle{ \alpha_1,\alpha_2,...\alpha_n}\) ma długość nie większą niż 1. Udowodnij że w wyrażeniu
\(\displaystyle{ \beta = \pm \alpha_1\pm \alpha_2 \pm...\pm \alpha_n}\)
można tak dobrać znaki aby \(\displaystyle{ |\beta| \le \sqrt{2}}\)
Dumel
[Planimetria] Długość sumy wektorów

63.
Dana jest liczba naturalna \(\displaystyle{ n}\). Udowodnić że pewna wielokrotność liczby \(\displaystyle{ n}\) ma w systemie 7-kowym postać \(\displaystyle{ 333 \ldots 300 \ldots 0}\)
matti44
[Kombinatoryka] Zasada szufladkowa dirichleta
Problem rozwiązany przez użytkownika Ponewor.
64. Mix 30, Iteracje
Niech \(\displaystyle{ f}\) będzie funkcją \(\displaystyle{ f: [0,1] \mapsto [0,1]}\) ciągła i taką, że dla dowolnego \(\displaystyle{ x \in [0, 1]}\) w ciągu iteracji \(\displaystyle{ x, f(x), f( f(x) ), …}\) występuje element \(\displaystyle{ 0}\). Czy z tego wynika, iż \(\displaystyle{ \underbrace{f (f (f( \ldots )))}_{n} \equiv 0}\) dla jakiegoś \(\displaystyle{ n}\) ?
mol_ksiazkowy
[MIX] Mix matematyczny (30)
Ukryta treść:    
65. Zestaw od Iwana, Czwórki i kwadraty
Znaleźć cztery takie liczby naturalne, aby suma każdych dwóch spośród nich była pełnym kwadratem. (Jest to uogólnienie zadania Diofantosa dla trzech liczb, - wtedy można wziąść np. \(\displaystyle{ 41, 80, 320}\)).
mol_ksiazkowy
[MIX][Teoria liczb] Zestaw od Iwana

66. Mix zestaw 50 zadań, Problem 34
Znajdź wszystkie trójki liczb naturalnych \(\displaystyle{ (x,y,u)}\) spełniające układ równań :
\(\displaystyle{ \begin{cases}x+y=u+12\\ x^{5}+y^{5}=u^{5}+12\end{cases}}\)
wiedzmac
[MIX] Zestaw pięćdziesięciu zadań przed II etapem OM

67. Mix 9, Czworokąty
Przekątne czworokąta \(\displaystyle{ ABCD}\) przecinają się w punkcie \(\displaystyle{ O}\). Niech punkty \(\displaystyle{ P, Q, R, S}\) będą rzutami punktu \(\displaystyle{ O}\) odpowiednio na boki \(\displaystyle{ AB, BC, CD, AD}\). Udowodnić, że czworokąt \(\displaystyle{ ABCD}\) jest wpisany w okrąg wtedy i tylko wtedy, gdy w czworokąt \(\displaystyle{ PQRS}\) da się wpisać okrąg.
limes123
[MIX] Mix matematyczny (9)

68. Układ w całkowitych
Rozwiązać:
\(\displaystyle{ \begin{cases} n+k=(NWD(n,k))^2\\k+m=(NWD(k,m))^2\\n+m=(NWD(m,n))^2 \end{cases}}\)
\(\displaystyle{ n, m, k = ?}\)
mol_ksiazkowy
[Teoria liczb] Układ równan;
Problem rozwiązany przez użytkownika mol_ksiazkowy.

69. Mix, Gra
Joasia i Onufry grają w następującą grę. Na tablicy są napisane liczby od 1 do 1000. Ruch polega na wymazaniu liczby, która nie jest dzielnikiem żadnej z wymazanych dotąd liczb. Przegrywa ten, kto nie może wykonać ruchu. Joasia zaczyna. Kto ma strategię wygrywającą ?
Swistak
[MIX] Warsztaty Matematyczne Staszica 2010
Problem rozwiązany przez użytkownika Ponewor.
70. Układ równań
Rozwiąż
\(\displaystyle{ \begin{cases}xy+\sqrt{2(x^{4}+y^{4})}=1\\ x^{2009}y^{2013}=\frac{2}{3^{2011}}\end{cases}}\)
darek20
[Równania] układ równań

71. Liczby na tablicy
na tablicy mamy \(\displaystyle{ n}\) liczb rzeczywistych. ruch polega na wybraniu dwóch liczb i zastąpieniu każdej z nich przez ich sumę. wyznaczyć wszystkie liczby \(\displaystyle{ n}\), dla których zawsze (tj dla dowolnych początkowych liczb) można osiągnąć \(\displaystyle{ n}\) równych liczb.
Dumel
[Kombinatoryka] liczby na tablicy (kombinatoryka)

72. Graf
Graf nazywamy \(\displaystyle{ k}\)-krytycznym, jeśli \(\displaystyle{ \chi(G)=k}\) oraz dla każdego \(\displaystyle{ v \in V(G)}\) jest \(\displaystyle{ \chi(G-v)< k}\). Wykazać, że dla każdego \(\displaystyle{ k}\)-krytycznego grafu G zachodzi \(\displaystyle{ \delta(G) \ge k-1}\)
nieOna3
Graf k-krytyczny

73. Ograniczenie maksimum
Niech \(\displaystyle{ a_{i}, i=1,2,3,\cdots,n}\) są liczbami naturalnymi takimi że \(\displaystyle{ ~ \frac{1}{a_{1}}+\frac{1}{a_{2}}+\cdots +\frac{1}{a_{n}}=1}\)
Wykaż że \(\displaystyle{ ~ max~(a_{1},a_{2},\cdots, a_{n})\leq n^{2^{n-1}}}\)
darek20
[Nierówności] wykazanie nierównosci z liczbami naturalnymi

74. Mix, Ciągi vs szeregi
Niech \(\displaystyle{ a_n}\) oznacza ilość tych ciągów "zero- jedynkowych" mających \(\displaystyle{ n}\) elementów, które nie zawierają sekwencji \(\displaystyle{ ...., 0, 1, 0, ....}\); zaś \(\displaystyle{ b_n}\) ilość tych, które nie zawierają sekwencji: \(\displaystyle{ ..., 0, 0, 1, 1, ....}\) ani \(\displaystyle{ ..., 1, 1, 0, 0, ....}\). Wykazać, iż \(\displaystyle{ b_{n+1}=2a_n}\) dla \(\displaystyle{ n \geq 2}\)
mol_ksiazkowy
[MIX][Ciągi][Analiza] Ciągi vs szeregi

75. Klub 444, runda piąta
Dane są liczby całkowite dodatnie \(\displaystyle{ a, b, c}\) takie, że \(\displaystyle{ ab|c(c^2-c+1)}\) i \(\displaystyle{ a+b|c^2 + 1}\). Pokazać, że zbiory \(\displaystyle{ \left\{ a,b\right\}}\) i \(\displaystyle{ \left\{ c, c^2-c +1\right\}}\) są równe.
Coach
[MIX][Klub 444] Runda piąta

76. Nierówność z modułami
Udowodnić w liczbach rzeczywistych, że:
\(\displaystyle{ \left| a\right| + \left| b\right| + \left| c\right| - \left| a+b \right| - \left| b+c \right| - \left| c+a \right| + \left| a+b+c \right| \ge 0}\)
Ukryta treść:    
skazy
[Nierówności] Suma modułów
Problem rozwiązany przez użytkowników timon92 i Ponewor.
77. Zadanie z Hong Kongu z 94
W turnieju udział bierze 10 zawodników, każda para uczestników gra ze sobą jeden raz. TRÓJKĄTEM będziemy nazywać trójkę uczestników: \(\displaystyle{ A, B, C}\), takich, że \(\displaystyle{ A}\) wygrał z \(\displaystyle{ B}\) , \(\displaystyle{ B}\) wygrał z \(\displaystyle{ C}\) , \(\displaystyle{ C}\) wygrał z \(\displaystyle{ A}\). Niech ponadto \(\displaystyle{ W_i}\) oznacza ilość gier wygranych przez \(\displaystyle{ i}\)-tego zawodnika, \(\displaystyle{ L_i}\)- ilość gier przegranych. Wiemy, że jeżeli \(\displaystyle{ i}\)-ty zawodnik wygrał z \(\displaystyle{ j}\)-tym, to \(\displaystyle{ L_i+W_j \ge 8}\).
Udowodnij, że w turnieju wystąpiło DOKŁADNIE 40 trójkątów.
jgarnek
[Kombinatoryka] Kombinatoryka z Hong Kongu 94'

78. Ciąg
Dany jest niemalejący ciąg liczb naturalnych \(\displaystyle{ a_n}\).Wykazać, że jeśli ciąg \(\displaystyle{ \frac{n}{a_n}}\) jest nieograniczony, to wśród jego wyrazów tego ciągu jest nieskończenie wiele liczb całkowitych
mol_ksiazkowy
sprytny ciąg
Problem rozwiązany przez użytkownika mol_ksiazkowy.
79. Szacowanie pierwiastka
Niech \(\displaystyle{ x \neq 0}\) będzie rzeczywistym pierwiastkiem równania \(\displaystyle{ ax^2+bx+c=0}\). Ponadto ,niech \(\displaystyle{ a, b}\) i \(\displaystyle{ c}\) będą całkowite spełniające nierówność \(\displaystyle{ \vert a\vert+\vert b\vert+\vert c\vert >1}\). Wykaż że \(\displaystyle{ \vert x\vert\geq \frac{1}{\vert a\vert+\vert b\vert+\vert c\vert-1}}\).

[Wielomiany] funkcja kwadratowa
marek12

80. Pochodna
Niech \(\displaystyle{ f(x)=\frac{2x}{1+e^x}}\). Udowodnij że dla każdego \(\displaystyle{ n}\) \(\displaystyle{ f^{(n)}(0)}\) jest całkowite
marek12
n-ta pochodna całkowita

81. Płaszczyzna i punkty
Danych jest \(\displaystyle{ 2n}\) różnych punktów: \(\displaystyle{ n}\) białych i \(\displaystyle{ n}\) czarnych. Żadne trzy z nich nie są współliniowe. Udowodnić, ze można tak narysować \(\displaystyle{ n}\) odcinków o końcach w danych \(\displaystyle{ 2n}\) punktach, aby były one różnokolorowe i aby nie przecinały się one ze sobą.
mol_ksiazkowy
Płaszczyzna i punkty
Problem rozwiązany przez użytkownika Ponewor.
82. Ciekawa teoria liczb
Liczby całkowite dodatnie \(\displaystyle{ a, b}\) spełniają następujący warunek:
\(\displaystyle{ 2ab-1|4a^4-2a^2+1}\).
Należy udowodnić, że ten warunek pociąga za sobą:
\(\displaystyle{ \sqrt{ab-1}}\) jest liczbą całkowitą.
Xmas11
[Teoria liczb] Ciekawa teoria liczb

83. Mix, Król i wieża
Pewne pola na szachownicy zamalowano tak, że król nie może dotrzeć od lewego do prawego jej brzegu po polach zamalowanych. Udowodnić, że po nie zamalowanych polach może od dolnego do górnego brzegu szachownicy dotrzeć wieża.
mol_ksiazkowy
[MIX][Kombinatoryka] mix (15) (tylko dla malarzy)
Problem rozwiązany przez użytkownika limes123.
84. Istnienie funkcji
Czy istnieje funkcja \(\displaystyle{ f: \RR^+\to \RR^+}\) taka że \(\displaystyle{ f(y) > (y - x) (f(x))^2}\) dla \(\displaystyle{ x, y \in \RR^+}\) gdzie \(\displaystyle{ y > x > 0}\)?
Ukryta treść:    
darek20
[Funkcje][Nierówności] istnienie funkcji?

85. Trójka
Jest \(\displaystyle{ n+1}\) liczb naturalnych mniejszych od \(\displaystyle{ 2n}\). Wykazać, że można wybrać z nich trzy takie, aby jedna z nich była sumą pozostałych dwóch.
hubertwojtowicz
Wykaż, że można wybrać
Problem rozwiązany przez użytkownika Ponewor.
86. Suma logarytmów
Jak policzyć tę sumę:
\(\displaystyle{ \frac{1}{\log 2} + \frac{1}{(\log 2)(\log 3)} + \frac{1}{(\log 2)(\log 3)(\log 4)} + ...}\)
marek12
suma logarytmów

87. Mix 26
Na płaszczyźnie leży \(\displaystyle{ 2n+3}\) punktów, tak iż żadne trzy nie leżą na jednej prostej i żadne cztery na jednym okręgu. Wykaż, że wtedy można narysować okrąg na którym są trzy z tych punktów, tak iż z pozostałych \(\displaystyle{ 2n}\) punktów dokładnie \(\displaystyle{ n}\) leży wewnątrz tego okręgu a \(\displaystyle{ n}\) pozostałych na zewnątrz tegoż okregu.
mol_ksiazkowy
[MIX] Mix matematyczny (26)
Problem rozwiązany przez użytkownika Ponewor.
88. Generator modulo \(\displaystyle{ p}\)
Niech \(\displaystyle{ p}\) będzie liczbą pierwszą dającą przy dzieleniu przez 20 resztę 11 lub 19. Udowodnić, że istnieją 3 kolejne liczby całkowite, które są generatorami modulo \(\displaystyle{ p}\)
KPR
[Teoria liczb] Urodziwe liczby pierwsze

89. Mix 14, Trzy punkty
Dany jest okrąg \(\displaystyle{ ABC}\) i okrąg opisany na nim \(\displaystyle{ o}\). Styczna do \(\displaystyle{ o}\) w punkcie \(\displaystyle{ A}\) przecina prostą \(\displaystyle{ BC}\) w punkcie \(\displaystyle{ D}\). Prosta prostopadła do \(\displaystyle{ BC}\) w punkcie \(\displaystyle{ B}\) przecina symetralną \(\displaystyle{ AB}\) w punkcie \(\displaystyle{ E}\) a prosta prostopadła do \(\displaystyle{ BC}\) w punkcie \(\displaystyle{ C}\) przecina symetralną \(\displaystyle{ AC}\) w punkcie \(\displaystyle{ F}\). Udowodnij że \(\displaystyle{ D, E, F}\) leżą na jednej prostej
Menda
[MIX] Mix matematyczny (14)
Problem rozwiązany przez użytkownika kaszubki.
90. Iloczyn
Wartość wyrażenia \(\displaystyle{ (2^{\frac{1}{3}}-1)^{\frac{1}{3}}}\) da się przedstawić w formie \(\displaystyle{ a^{\frac{1}{3}}+b^{\frac{1}{3}}+c^{\frac{1}{3}}}\), gdzie \(\displaystyle{ a, b, c}\) liczby wymierne. Oblicz \(\displaystyle{ abc}\)
darek20
wartość iloczynu
Problem rozwiązany połowicznie przez użytkownika rochaj.
91. Nietypowa podzielność
Dla jakich par \(\displaystyle{ (m,n)}\) gdzie \(\displaystyle{ m \in \ZZ \ , n \in \ZZ}\) zachodzi:
\(\displaystyle{ \frac{n^3+1}{mn-1} \in \ZZ}\) ?
Ukryta treść:    
mol_ksiazkowy
pary liczb
Problem rozwiązany przez użytkownika Ponewor.
92. Lemat o trójkącie
Dany jest trójkąt ostrokątny \(\displaystyle{ ABC}\) oraz punkt \(\displaystyle{ P}\), w jego wnętrzu taki, że kąty \(\displaystyle{ CAP}\) i \(\displaystyle{ CBP}\) są równe. Niech \(\displaystyle{ K}\) i \(\displaystyle{ L}\) są rzutami punktu \(\displaystyle{ P}\) na boki \(\displaystyle{ AC}\) i \(\displaystyle{ BC}\). Wykazać że symetralna odcinka \(\displaystyle{ KL}\) dzieli na pół bok \(\displaystyle{ AB}\)
Ukryta treść:    
mol_ksiazkowy
maly lemat
Problem rozwiązany przez użytkownika timon92.
93. Mix, Punkty wymierne
Udowodnij, że równanie ma \(\displaystyle{ y^2=x^3+x+1370^{1370}}\) ma co najmniej 6 rozwiązań w \(\displaystyle{ \mathbb{Q}}\)
robin5hood
[MIX] Mix robina

94. Następna nierówność
Niech \(\displaystyle{ a,b,c \geq 0 , \ a^{2}+b^{2}+c^{2}=2}\). Wykaż:
\(\displaystyle{ \frac{1+a^{2}b}{a^{2}+b^{2}}+\frac{1+b^{2}c}{b^{2}+c^{2}}+\frac{1+c^{2}a}{c^{2}+a^{2}}\geq 3}\)
darek20
[Nierówności] wykazanie nierówności III

95. Szukane funkcje
Wyznaczyc wszystkie funkcję ciągłe \(\displaystyle{ f:[0, \infty)\to\mathbb{R}}\) takie ze \(\displaystyle{ \displaystyle\lim_{x\to \infty} f(x)=0}\) oraz \(\displaystyle{ f(x)+3f(x^2)=5f(x^3+1)}\) dla \(\displaystyle{ x\geq 0.}\)
darek20
[Planimetria] Okręgi w trójkącie prostokątnym

96. Specjalna rekurencja
Niech \(\displaystyle{ f : \mathbb{Z^* \times Z} \to \mathbb{Z}}\) będzie funkcją spełniającą:
1. \(\displaystyle{ f(0, k) = 1}\) dla \(\displaystyle{ k = 0, 1}\)
2. \(\displaystyle{ f(0, k) = 0}\) dla \(\displaystyle{ k \not\in \{0,1\}}\)
3. \(\displaystyle{ f(n, k) = f(n - 1, k) + f(n - 1, k - 2n)}\) dla dowolnych \(\displaystyle{ n \ge 1, k}\)
wyznacz
\(\displaystyle{ \sum_{k=0}^{ {n+1 \choose 2} }f(n,k)}\)

(\(\displaystyle{ \ZZ*}\) - całkowite nieujemne)
Dumel
[Kombinatoryka] suma do wyznaczenia i funkcja

97. Złożenie funkcji
Rozstrzygnąć czy istnieje funkcja różniczkowalna \(\displaystyle{ f: \mathbb{R} \longrightarrow \mathbb{R},}\) taka że \(\displaystyle{ (f \circ f)(x) = \sin x.}\)
marek12
[Równania funkcyjne] złozenie funkcji

98. Porównanie i równość
Niech \(\displaystyle{ x >0}\) i \(\displaystyle{ y >0}\) :
\(\displaystyle{ (\frac{1}{3} (x+ (xy)^{\frac{1}{2}}+ y))^{\frac{1}{3}} = (\frac{1}{2}(x^{\frac{2}{3}}+y^{\frac{2}{3}}))^{\frac{1}{2}}}\)
Czy \(\displaystyle{ x=y}\) ?
mol_ksiazkowy
[Nierówności] Dwie liczby
Problem rozwiązany przez użytkownika Ponewor.
99. Potrójna podzielność
Znaleźć wszystkie liczby naturalne \(\displaystyle{ a,b,c}\) takie że:
\(\displaystyle{ \begin{cases} 1+a|b^2+c^2\\1+b|c^2+a^2\\1+c|a^2+b^2\end{cases}}\)
rochaj
[Teoria liczb] układ z podzielnością

100.
Znajdź wszystkie czwórki liczb naturalnych \(\displaystyle{ (a,b,c,d)}\) takich, że \(\displaystyle{ \\ \begin{cases} a | bcd+1 \\ b | cda+1 \\ c | dab +1 \\ d | abc+1 \end{cases}.}\)
czekoladowy
[Teoria liczb] Układ podzielności
Problem rozwiązany przez użytkownika Ponewor.
101. Własność funkcji
Znaleźć wszystkie funkcje rzeczywiste \(\displaystyle{ f\,:\,\mathbb{R}_+\to\mathbb{R}_+}\) spełniające równanie funkcyjne (tj. \(\displaystyle{ \,f(x)\,f(y)\,=\,f\big(y\,f(x)\big)\,}\) ).
Uwaga:, tu już nie wymagamy, aby funkcja była surjekcją, a jedynie by liczby dodatnie przekształcała na dodatnie!
SirGeorge
[Równania funkcyjne] Równanie funkcyjne (surjekcja)
Ostatnio zmieniony 22 lip 2016, o 23:24 przez mol_ksiazkowy, łącznie zmieniany 17 razy.
Awatar użytkownika
mol_ksiazkowy
Użytkownik
Użytkownik
Posty: 11263
Rejestracja: 9 maja 2006, o 12:35
Płeć: Mężczyzna
Lokalizacja: Kraków
Podziękował: 3140 razy
Pomógł: 747 razy

[MIX] Nierozwiązane problemy

Post autor: mol_ksiazkowy »

Są one i z takiej tematyki:
algebra
geometria
Nierówności;
kombinatoryka
probablistyka, grafy
itd. inne.

A więc:
Brak wiec zadań z działów: Analiza funkcjonalna, Algebra abstrakcyjna, Równania różniczkowe itd.

Niektóre problemy są jakoś tam częściowo przybliżone, oraz są i źródła...

Ewentualne błędy (np. wadliwe linki) > pw.


Nierozwiązane problemy III

stan: 34/101 (aktualnie), tj. ok. 33, 6 %

Zadania:

1. Mix Robina, piętrowe potęgi
Znajdź najmniejszą liczbę naturalną \(\displaystyle{ m}\) spełniająca tę nierówność
\(\displaystyle{ \overbrace { 100^{100^{100^{.^{.^{.^{100}}}}}}}^m > \overbrace {3^{3^{3^{.^{.^{.^{3}}}}}}}^{100}}\)
robin5hood
[MIX] Mix robina
Problem rozwiązany przez użytkownika marcin7Cd.


2. Iteracja
Niech \(\displaystyle{ f(x)=x^{2}+bx+1}\) gdzie \(\displaystyle{ b\in (2;\frac{7}{2})}\)
Rozwiąż nierówność \(\displaystyle{ f[f(x)]> x}\)
marek12
[MIX] różne "na wakacje od dziś" II

3. Mix 12 zadań, suma kwadratem
Znaleźć wszystkie liczby \(\displaystyle{ a, b{\in}N}\) takie że \(\displaystyle{ a^b+b^a}\) jest kwadratem liczby.
rochaj

[MIX] Dwanaście zadań

4. Szczególna trójka
Dla jakich \(\displaystyle{ x, y, z \in \RR}\)

\(\displaystyle{ \{ x, y , z \} = \{ \frac{x-y}{y-z}, \frac{y-z}{z-x}, \frac{z-x}{x-y} \}}\)

o ile \(\displaystyle{ x \neq y \neq z \neq x}\) ?
mol_ksiazkowy
Równość zbiorów
Problem rozwiązany przez użytkownika mol_ksiazkowy.


5. Który współczynnik ?
Czy największym współczynnikiem \(\displaystyle{ (1+2x+3x^2+4x^3)^n}\) dla \(\displaystyle{ n>1}\) jest ten przy \(\displaystyle{ x^{2n}}\) ?

Bo np. \(\displaystyle{ (1+2x+3x^2+4x^3)^2= 1 +4x+ 10x^2+ 20x^3 + 25x^4 + 24x^5+ 16x^6}\) itd.
mol_ksiazkowy
Największy współczynnik
Ukryta treść:    


6. Bilard
W układzie współrzędnych rysujemy parabolę o równaniu \(\displaystyle{ y=x^2}\). Niech \(\displaystyle{ \alpha}\) będzie obszarem w I ćwiartce pomiędzy parabolą a osią \(\displaystyle{ OX}\). W obszarze tym porusza się kula bilardowa (jako punkt materialny) odbijając się od brzegu \(\displaystyle{ \alpha}\) zgodnie z regułą "kąt padania=kąt odbicia". Dowieść, że kula odbije się od brzegu \(\displaystyle{ \alpha}\) skończenie wiele razy.
patry93
[Kombinatoryka] Kula bilardowa odbija się od paraboli
Ukryta treść:    
Problem rozwiązany link .


7. Ciekawe potęgi
Czy istnieją takie liczby nieparzyste \(\displaystyle{ a,b\geqslant3, a < b}\), dla których równanie:

\(\displaystyle{ (2^a-1)^k = 2^b-1}\)

będzie miało naturalne rozwiązanie \(\displaystyle{ k}\). Wydaje się że powinno pójść Fermatem...,
JPlenti
Chyba Fermat...
Problem rozwiązany przez użytkownika mol_ksiazkowy.


8. Dwa ciągi
Mamy dane 2 ciągi \(\displaystyle{ a_1, a_2, ..., a_n}\) oraz \(\displaystyle{ b_1, b_2, ..., b_n}\) liczb
a) całkowitych
b) rzeczywistych
takie, że ciągi wartości wielomianów Newtona \(\displaystyle{ n}\) zmiennych o stopniach \(\displaystyle{ 1, 2, ..., n}\) są dla tych ciągów równe. Dla jakich \(\displaystyle{ n}\) ciąg \(\displaystyle{ b}\) musi być permutacją ciągu \(\displaystyle{ a}\).
Swistak
[Wielomiany] Wielomiany Newtona

9. Szczególne wielomiany
Wyznaczyć wszystkie wielomiany \(\displaystyle{ f}\) o współczynnikach całkowitych takie, że \(\displaystyle{ f(p)|2^p - 2}\) dla każdej liczby pierwszej \(\displaystyle{ p}\).
Coatch
[MIX][Klub 444] Runda druga
Problem rozwiązany przez użytkownika Marcin7Cd.


10. Minimum w trójkącie
Mamy trójkąt \(\displaystyle{ ABC}\), znaleźć minimum wyrażenia
\(\displaystyle{ (1+\cos^2(A))(1+\cos^2(B))(1+\cos^2(C))}\)
darek20
minimum w trójkącie

11. Ułamek będący kwadratem
Czy liczba \(\displaystyle{ \frac{2m(2m-1)}{2n(2n-1)}}\) może być kwadratem liczby naturalnej dla pewnych \(\displaystyle{ m,n\in\mathbb N^+,\ n\neq m\ ?}\)
Ukryta treść:    
Jan Kraszewski
Czy liczba może być kwadratem?
Problem rozwiązany połowicznie przez użytkownika mol_ksiazkowy.


12. Zwijanie iloczynu
Wyznaczyć wzór jawny:

\(\displaystyle{ \prod_{n=1}^{\infty} \frac{a^k+a^n}{1+a^n}}\)

dla:
a) \(\displaystyle{ a,k\in\mathbb{N}}\),
b) \(\displaystyle{ a,k\in\mathbb{Z}}\),
c*) \(\displaystyle{ a,k\in\mathbb{R}}\).
JakimPL
[Analiza] Produkt do wyznaczenia

13. Kółka zainteresowań
Danych jest \(\displaystyle{ 3n}\), ludzi \(\displaystyle{ n=1,2,3,\ldots}\) oraz kółka zainteresowań. W każdym kółku jest nieparzysta liczba członków, a część wspólna dowolnych \(\displaystyle{ 2^{n-1}+1}\) kółek zawiera parzystą liczbę członków. Udowodnić, że kółek jest nie więcej niż \(\displaystyle{ 2^n+n2^{n-1}}\).
marek12
[Kombinatoryka] kółka zainteresowań

14. Oszacowanie
Niech \(\displaystyle{ x_1,x_2,\dots,x_n>0}\) będą takie że \(\displaystyle{ x_1x_2\cdots x_n = 1}\). Pokaż że \(\displaystyle{ \frac{x_k}{k + x_1 + x_2 + \dots + x_k} \geq 1 - \frac{1}{\sqrt[n]{2}}}\) dla pewnego \(\displaystyle{ k \in \{1,2,\dots,n\}.}\)
robin5hood
[Nierówności] wykazanie nierówności

15. Ciąg Fibonacciego i nierówność
\(\displaystyle{ F_1 = F_2 = 1, F_{n+2 }= F_{n+1} + F_n; n\geq 1.}\) Dla każdego \(\displaystyle{ n\geq 2}\) i dla każdego rzeczywistego \(\displaystyle{ x}\) wykazać nierówność

\(\displaystyle{ \sum_{k=1}^n|x-k|F_k\geq F_{n+2}+F_n-n-1}\).
marek12
[Ciągi] ciąg Fibonacciego

16. Iteracje
Dana jest ustalona para \(\displaystyle{ (s,t)}\) liczb całkowitych, \(\displaystyle{ s \neq 0 \neq t}\). Mając inna parę liczb całkowitych \(\displaystyle{ (x,y)}\) zastępujemy ją wg schematu \(\displaystyle{ (x,y) \mapsto (x+t,y-s)}\). Powiemy że para \(\displaystyle{ (x,y)}\) jest szczególna , gdy po skończonej liczbie kroków (być może równej zeru) uzyskamy pewną parę \(\displaystyle{ (x^\prime, y^\prime)}\) liczb które nie są względnie pierwsze.
1) Zbadaj czy \(\displaystyle{ (s, t)}\) jest szczególna ?
2) Wykaż że dla dowolnej \(\displaystyle{ (s,t)}\) istnieje para \(\displaystyle{ (x,y)}\) która nie jest szczególna
mol_ksiazkowy
[Teoria liczb] Dobra para

17. Mix 30, Wyrażenia kwadratami
Udowodnić, że istnieje nieskończenie wiele par \(\displaystyle{ (t, v)}\) liczb naturalnych o tej własności, iż \(\displaystyle{ tv - 2\sqrt{v^2 + t^2} +2}\) jest kwadratem liczby całkowitej.
b) to samo: dla par \(\displaystyle{ (u, z)}\) i wyrażenia \(\displaystyle{ \frac{21u + 6z - 20}{7}}\)
mol_ksiazkowy
[MIX] Mix matematyczny (30)
Problem rozwiązany połowicznie przez użytkownika mol_ksiazkowy.


18. Zestaw od Iwana, Trójki
Wyznaczyć wszystkie trójki \(\displaystyle{ ( x, y, z )}\) liczb całkowitych takie, że liczby \(\displaystyle{ x^2 + y, \ y^2 + z, \ z ^2 + x}\) są kwadratami liczb całkowitych.
mol_ksiazkowy
[MIX][Teoria liczb] Zestaw od Iwana
Ukryta treść:    


19. Następna nierówność
Niech \(\displaystyle{ a,b>0;a^{3}+b^{5}\leq a^{2}+b^{2}.}\) Pokaż że \(\displaystyle{ b-\frac{1}{a^{2}+b^{2}}\leq \frac{1}{2}}\).
rochaj
[Nierówności] Nierówność z dwiema niewiadomymi

20. Sumy kwadratów
Niech \(\displaystyle{ K=\{n|n=a^2+b^2, a, b, n \in \mathbb{N}, a , b >0\}.}\) Znaleźć wszystkie liczby całkowite \(\displaystyle{ m}\) takie, że \(\displaystyle{ \{m, m+1, m+2\} \subset K.}\)
rochaj
[Teoria liczb] podzbiór zbioru
Problem rozwiązany połowicznie przez użytkownika Martingale.


21. Mix Zadania różne; Dziwna funkcja
Wyznaczyć takie \(\displaystyle{ f: \RR \to \RR}\) różniczkowalne, nietożsamościowo zerowe i takie, że:

\(\displaystyle{ f(x)= x f'(\frac{x}{\sqrt{3}})}\) dla \(\displaystyle{ x \in \RR.}\)
mol_ksiazkowy
[MIX] Zadania różne
Ukryta treść:    


22. Niezmiennik ciągu
Z danego ciągu liczb \(\displaystyle{ 1, 2, ..., 1968, 1969}\) możemy usunąć dwie liczby, dopisując jednocześnie na końcu ciągu wartość bezwzględną ich różnicy. Czy przez wielokrotne zastosowanie tej operacji można otrzymać ciąg złożony z samych zer?

Wydaje mi się, że niezmiennikiem będzie w tym przypadku nieparzystość tego ciągu, lecz czy są inne niezmienne w tym zadaniu?
lordbross
Niezmiennik ciągu
Ukryta treść:    


23. Układ nierówności
Niech \(\displaystyle{ a>b>c>1}\). Znajdź rozwiązania w liczbach dodatnich:

\(\displaystyle{ \begin{cases} ax>y+z \\ by>x+z \\ cz>x+y \end{cases}}\)
Tifulo
[Analiza] Jakie warunki muszą zajść by..

24. Nietrywialne uogólnienie
W jednej z kategorii na tegorocznym Kangurze, pojawił się taki o to problem:
Ile liczb całkowitych dodatnich będących wielokrotnościami liczby \(\displaystyle{ 2013}\) ma dokładnie \(\displaystyle{ 2013}\) różnych dodatnich dzielników (do dzielników liczby zaliczamy \(\displaystyle{ 1}\) i tę liczbę) ?


Otóż ja podbijam i stawiam ogólniejszy problem:
Dana jest liczba naturalna \(\displaystyle{ n > 1}\), oraz jej rozkład na czynniki pierwsze \(\displaystyle{ n= \prod_{i=1}^{m}p_{i}^{\alpha_{i}}}\). Ile liczb całkowitych dodatnich będących wielokrotnościami liczby \(\displaystyle{ n}\) ma dokładnie \(\displaystyle{ n}\) różnych dodatnich dzielników?

Ukryta treść:    
Ponewor
[Teoria liczb] Wielokrotność i suma dzielników
Problem rozwiązany przez użytkownika Marcin7Cd.


25. Własność ciągu Fibonacciego
Udowodnić, że jeśli dodatnie całkowite \(\displaystyle{ n}\) NIE jest postaci \(\displaystyle{ 2^y3^x}\) (gdzie \(\displaystyle{ x}\) i \(\displaystyle{ y}\) są całkowite nieujemne), to \(\displaystyle{ F_n}\) ma co najmniej jeden dzielnik pierwszy postaci \(\displaystyle{ 4k+1}\) (gdzie \(\displaystyle{ k}\) jest całkowite dodatnie).

no i \(\displaystyle{ F_n}\) to \(\displaystyle{ n}\)-ty wyraz ciągu Fibonacciego.
jakub jabulko
[Teoria liczb] Dzielniki pierwsze, ciąg Fibonacciego

26. Mix Nierówności różne; Nieistniejąca funkcja
Wykazać, że nie istnieje \(\displaystyle{ f: \RR_{+} \to \RR_{+}}\) taka iż

\(\displaystyle{ f \left( x \right) ^2 \geq f \left( x+y \right) \left( f \left( x \right) + y \right)}\)

dla \(\displaystyle{ x, y \in \RR_{+}.}\)
mol_ksiazkowy
[MIX][Nierówności] Nierówności różne
Ukryta treść:    
Problem rozwiązany link .


27. NWD i NWW elementarnie
Nie korzystając z pojęcia liczb pierwszych i własności z nimi związanych (ale tylko z nimi, typu rozkład na czynniki pierwsze) oraz pojęcia względnej pierwszości, pokazać że:

dla dowolnych liczb naturalnych \(\displaystyle{ a,b,c}\) zachodzi \(\displaystyle{ abc=[a,b,c](ab,ac,bc)=(a,b,c)[ab,ac,bc]}\) oraz \(\displaystyle{ [a,b,c](a,b,c)\mid abc.}\)

Dodatkowo - pokazać, że \(\displaystyle{ abc=[a,b,c](a,b,c) \Leftrightarrow (a,b)=(a,c)=(b,c)=1}\).
tatteredspire
Iloczyn trzech liczb naturalnych

28. Diofantos, trzy zmienne
Rozwiązać równanie w zbiorze liczb naturalnych

\(\displaystyle{ (a+2)(b+2)(c+2) = (a+b+c+2)^2}\).
darek20
[Teoria liczb] Równanie w liczbach naturalnych
Problem rozwiązany połowicznie przez użytkownika JakimPL.


29. Pokrycie wielokąta
Dla każdej trójki kolejnych wierzchołków wielokąta wypukłego narysowano okrąg przechodzący przez te wierzchołki. Udowodnić, że ten spośród otrzymanych okręgów, który ma największy promień zawiera cały wielokąt.

Uzasadnić, ze założenie o wypukłości jest tu istotne.
mol_ksiazkowy
Pokrycie wielokąta

30. Mix dla szachistów; Hetmany
(M. Gardner) Ustawić 20 hetmanów na szachownicy \(\displaystyle{ 8 \times 8}\) w taki sposób, aby każdy hetman mógł zbić (wykonując jeden ruch) dokładnie cztery z pozostałych 19 hetmanów
mol_ksiazkowy
[MIX][Kombinatoryka] MIX dla szachistów
Problem rozwiązany przez użytkownika HuBson.


31. Totolotek
Ile jest 6-elementowych podzbiorów zbioru \(\displaystyle{ \left\{ 1,2,3,...,49\right\}}\), takich, że nie ma w nim żadnych dwóch kolejnych liczb.
Ukryta treść:    
porfirion
[Kombinatoryka] Duży lotek
Problem rozwiązany przez użytkownika fon_nojman.


32. Mix Zestaw mola (końcówka); liczba taksówkowa
Czy z tożsamości Ramanujana:

\(\displaystyle{ (3a^2 + 5ab - 5b^2)^3 + (4a^2 - 4ab - 5b^2)^3 = (- 5a^2 + 5ab + 3b^2)^3 + (6a^2 - 4ab + 4b^2 )^3}\)

można uzyskać rozkład \(\displaystyle{ 1729 = 1^3 + 12^3 = 9^3 + 10^3}\) (liczba Hardy’ego ) znajdując w niej \(\displaystyle{ a}\) i \(\displaystyle{ b}\) ?
mol_ksiazkowy
[MIX] Zestaw mola (końcówka)
Ukryta treść:    
Problem rozwiązany przez użytkownika mol_ksiazkowy .


33. Nierówność z mantysą
Pokaż że dla każdej liczby naturalnej \(\displaystyle{ n}\) zachodzi \(\displaystyle{ \frac{1}{2n}< \{ \sqrt{7}n \} < 1 - \frac{1}{6n}}\)
rochaj
[Nierówności] nierówność w liczbach naturalnych 2
Problem rozwiązany przez użytkownika marcin7Cd .


34. Symetrie i zbiory
Czy istnieje figura (zbiór):
a) skończona (której promień jest skończony) o więcej niż 1 środku symetrii
b) nieskończona o \(\displaystyle{ n}\) środkach symetrii, gdzie \(\displaystyle{ n>1}\)
c) czy w innej niż euklidesowej metryce jest to możliwy któryś z powyższych przypadków.
exupery
środki symetrii

35. Szacowanie pochodnej
Niech \(\displaystyle{ f(x)}\) będzie wielomianem stopnia \(\displaystyle{ n}\). Pokaż że \(\displaystyle{ \max_{x\in [-1,1]}|f'(x)| \le n^2\max_{x\in [-1,1]}|f(x)|}\).
rochaj
[Analiza][Nierówności][Wielomiany] Pochodna wielomanu
Problem rozwiązany przez użytkownika Ponewor.


36. Gra z pionkami
Mamy sobie planszę \(\displaystyle{ 4n \times 4n}\) i ustawiamy na niej maksymalną liczbę pionków tak, aby w żadnym wierszu ani kolumnie nie stały 2 pionki. Każdego pionka chcemy przesunąć na którąś z dwóch przekątnych, ale nie obchodzi nas to, czy będzie on na jednej, czy na drugiej przekątnej. Ponadto na jednym polu może stać wiele pionków.

Ruch polega na przesunięciu pionka na sąsiednie pole.

Jaka jest maksymalna możliwa liczba ruchów potrzebna do przesunięcia wszystkich pionków na przekątne ?
kaszubki
[Kombinatoryka] Plansza 4n x 4n

37. Mix, turniej
(Musztari) W turnieju szachowym każdy z uczestników rozegrał po jednej partii ze wszystkimi pozostałymi. Wykaż, że uczestników turnieju można tak ponumerować, że żaden z nich nie przegrał partii z graczem o numerze większym o 1 od jego numeru.
mol_ksiazkowy
[MIX][Kombinatoryka] MIX dla szachistów
Problem rozwiązany przez użytkownika porfirion.


38. Mix Ciągi vs szeregi; rekurencja
Ciąg liczb dodatnich \(\displaystyle{ a_0, a_1, a_2, a_3, ...}\) spełnia zależność:
\(\displaystyle{ 2^{n+1}(a_{n-1}-a_n)=a_n^2}\) dla \(\displaystyle{ n \geq 1}\).
Wykazać zbieżność i obliczyć \(\displaystyle{ \lim a_n}\) (w zależności od wyrazu początkowego \(\displaystyle{ a_0}\)).
mol_ksiazkowy
[MIX][Ciągi][Analiza] Ciągi vs szeregi

39. Potęga w zbiorze
Niech \(\displaystyle{ m \ge 2, m\in \NN}\). Znaleźć najmniejszą liczbę całkowitą \(\displaystyle{ n>m}\) taką, że dla dowolnego podziału zbioru \(\displaystyle{ \{m,m+1,...,n\}}\) na dwa podzbiory, przynajmniej jeden zawiera trzy liczby \(\displaystyle{ a,b,c}\) takie, że \(\displaystyle{ c=a^b}\)
ares41
[MIX][Teoria liczb][Równania funkcyjne] 10 zadań

40. Niewymierność w ułamku
Usunąć niewymierność z mianownika \(\displaystyle{ \frac{1}{\sqrt{2}+\sqrt[3]{3}+\sqrt[4]{5}}}\) (nie chodzi o trywialne \(\displaystyle{ \frac{\frac{1}{\sqrt{2}+\sqrt[3]{3}+\sqrt[4]{5}}}{1}}\) itp.)
metalknigh
sprowadzanie mianownika do postaci wymiernej
Problem rozwiązany przez użytkownika fon_nojman.


41. Graf Petersena
Dany jest graf \(\displaystyle{ G = (V,E)}\), gdzie \(\displaystyle{ V = \left\{\left\{ i,j \right\} : i, j \in \left\{1,2,3,4,5\right\}\right\}}\), przy czym
\(\displaystyle{ uv \in E \Leftrightarrow u \cap v =\emptyset}\).

Udowodnij, że \(\displaystyle{ G}\) jest izomorficzny z grafem Petersena.

Nie potrafię narysować tego grafu, tzn. wychodzi mi graf pełny na 5 wierzchołkach, który nijak nie jest izomorficzny z grafem Petersena. (A może jest ?)
Darkwing Duck
Grafy eulerowskie i hamiltonowskie

42. Szacowanie dwa razy
Niech liczby naturalne \(\displaystyle{ m,n}\) spełniają nierówność \(\displaystyle{ \sqrt{2}+\sqrt{3}-\frac{m}{n}>0}\). Pokaż że \(\displaystyle{ \sqrt{2}+\sqrt{3}-\frac{m}{n}>\frac{1}{3mn^3}.}\)
rochaj
[Nierówności] nierówność w liczbach naturalnych
Ukryta treść:    


43. Jeszcze raz ciąg Fibonacciego
Jak wykazać, że jeśli \(\displaystyle{ n\in\mathbb{N}}\) i \(\displaystyle{ 5n^2\pm 4}\) jest kwadratem liczby naturalnej, to \(\displaystyle{ n}\) jest liczbą Fibonacciego ?
kamiles
liczby Fibonacciego

44. Niewymierność inaczej
Udowodnij że liczba \(\displaystyle{ \sqrt{5}}\) jest liczbą niewymierną (stosując przy tym indukcję matematyczną).
waliant
udowodnij niewymiernosc
Ukryta treść:    


45. Nierówność z czterema zmiennymi
bardzo fajna nierówność, chętnie zobaczę Wasze rozwiązania, bo zapewne można ją ugryźć z wielu stron:
dla dodatnich \(\displaystyle{ a,b,c,d}\) o sumie równej \(\displaystyle{ 4}\):

\(\displaystyle{ a^2bc+b^2cd+c^2da+d^2ab \le 4}\)

proszę tylko nie piszcie: "jak się ujednorodni i wymnoży to pewnie wyjdzie"
Dumel
[Nierówności] nierówność z MEMO

46. Nieprzecinające się odcinki
Jaka jest liczba połączeń odcinkami w pary wierzcholkow \(\displaystyle{ 2n}\) -kąta wypukłego, tak żeby odcinki się nie przecinały ?
lightinside
Liczba połączeń w pary nieprzecinajace się odcinki

47. Dirichlet inaczej
Mamy dwa koncentryczne dyski, każdy podzielony na \(\displaystyle{ 200}\) sektorów pomalowanych dwoma kolorami. Na zewnętrznym dysku liczba sektorów każdego koloru jest taka sama. Pokazać, że można tak nałożyć dyski na siebie, by uzyskać co najmniej \(\displaystyle{ 50\%}\) zgodności kolorów.
pelas_91
[Zasada szufladkowa] Zgodność dysków

48. Średnia z minimów
Dana jest rodzina wszystkich \(\displaystyle{ k}\)-elementowych podzbiorów zbioru \(\displaystyle{ \left\{ 1,2,......,2n\right\}}\). Dla każdego takiego podzbioru \(\displaystyle{ A}\) rozważmy jego najmniejszy element. Wyznaczyć średnią wielkość najmniejszego elementu po wszystkich takich \(\displaystyle{ k}\)-elementowych podzbiorach.
willhelm
średnia wielkość w zadaniach kombinatorycznych
Problem rozwiązany przez użytkownika mol_ksiazkowy.


49. Trzy proste i trójkąt
Dane są trzy proste równoległe. Skonstruuj trójkąt równoboczny, którego każdy wierzchołek leży na innej prostej.
Tomasz Rużycki
T. równoboczny o wierzch. leżących na prost. równolegĹ
Problem rozwiązany przez użytkownika mol_ksiazkowy.


50. Inna suma potęg
Udowodnić, że każdą liczbę naturalną \(\displaystyle{ n}\) można przedstawić w postaci sumy takich liczb postaci \(\displaystyle{ 2 ^{i}3 ^{j}}\), że żadna z nich nie jest dzielnikiem którejś z pozostałych.
bakala12
[Teoria liczb] Przedstawienie liczby w postaci pewnej sumy
Ukryta treść:    
Problem rozwiązany przez użytkownika Msciwoj.


51. Część wspólna sfer
Przez każdy wierzchołek i środki wychodzących zeń krawędzi danego czworościanu prowadzimy sferę. Udowodnij, że otrzymane w ten sposób cztery sfery mają punkt wspólny.
matex_06
[Stereometria] punkt wspólny

52. Rekurencja i limes
Niech \(\displaystyle{ \begin{cases}a_{0}>0 \\ a_{n+1}=a_{n}-e^{\frac{-1}{a_{n}^{2}}}\end{cases}}\). Pokaż ze \(\displaystyle{ \lim_{n\to \infty }a_n^2\ln(n)=1}\).
rochaj
[Ciągi] 3 ciągi rekurencyjne

53. Cecha i potęga dwójki
Znaleźć największą liczbe \(\displaystyle{ k \in \mathbb{N}}\) taką że \(\displaystyle{ \lfloor (1+\sqrt{3})^n \rfloor}\) jest podzielne przez \(\displaystyle{ 2^k}\)
darek20
podłoga i podzielność
Ukryta treść:    
Problem rozwiązany przez użytkownika mol_ksiazkowy .


54. Wędrówki pająka
Mamy \(\displaystyle{ n}\) okregów przecinajacych sie w punkcie \(\displaystyle{ O}\). Pająk wychodzi z punktu \(\displaystyle{ X}\) na okregu \(\displaystyle{ S_1}\) i idzie do punktu \(\displaystyle{ Y}\) na \(\displaystyle{ S_2}\) w taki sposób, że \(\displaystyle{ XY}\) przechodzi przez punkt przecięcia okregow \(\displaystyle{ S_1}\) i \(\displaystyle{ S_2}\) różny od \(\displaystyle{ O}\). Udowodnij, ze pająk wróci do X po \(\displaystyle{ n}\) takich przejściach.
limes123
[Kombinatoryka] Pająk chodzi po okręgach

55. Minimum warunkowe
Niech \(\displaystyle{ 3x+2y+z=1 ~}\) oraz \(\displaystyle{ x,y,z}\) są rzeczywiste.
Znaleźć maximum \(\displaystyle{ ~ \sum_{cyc}\frac{1}{1+|x|}}\).
darek20
[Nierówności] Maximum sumy

56. Jakie liczby ?
Znaleźć wszystkie liczby pierwsze \(\displaystyle{ p}\) takie że \(\displaystyle{ \frac{p+1}{2}}\) oraz \(\displaystyle{ \frac{p^2+1}{2}}\) są liczbami pierwszymi.
rochaj
[Teoria liczb] Znalezienie liczb pierwszych
Ukryta treść:    


57. Mix 28, Rozkład na sumę
1. a) Znaleźć wszystkie liczby naturalne \(\displaystyle{ a}\) takie że \(\displaystyle{ a-1}\) jest sumą dwóch (niekoniecznie różnych) dzielników liczby \(\displaystyle{ a}\)
b) Znaleźć wszystkie liczby naturalne \(\displaystyle{ a}\) takie że \(\displaystyle{ a-1}\) jest sumą trzech (niekoniecznie różnych) dzielników liczby \(\displaystyle{ a}\)
c) Podać przykład liczby \(\displaystyle{ a}\), takiej że \(\displaystyle{ a-1}\) jest sumą czterech (niekoniecznie różnych) dzielników liczby \(\displaystyle{ a}\)
d) Czy prawdziwe jest twierdzenie: dla dowolnej liczby naturalnej \(\displaystyle{ n}\) istnieje tylko skończona ilość liczb \(\displaystyle{ a}\) takich , że \(\displaystyle{ a-1}\) jest sumą \(\displaystyle{ n}\) (niekoniecznie różnych) dzielników liczby \(\displaystyle{ a}\)
mol_ksiazkowy
[MIX] Mix matematyczny (28)
Problem rozwiązany połowicznie przez użytkownika Ahhaa.


58. Zamek
Sekretny zamek ma na wspólnej osi 4 tarcze z których każda jest podzielona na 5 sektorów z napisanymi na nich cyframi. Zamek otwiera się tylko w takim położeniu tarcz przy którym cyfry na nich tworzą określona liczbę czterocyfrową. Znaleźć prawdopodobieństwo tego że przy przypadkowym ustawieniu tarcz zamek będzie można otworzyć.
kapi92
Sekretny zamek

59. Złożenie funkcji
Czy istnieje taka para funkcji \(\displaystyle{ g,h:\mathbb{R} \to \mathbb{R}}\), że jedyną funkcją \(\displaystyle{ f:\mathbb{R} \to \mathbb{R}}\) taką, że \(\displaystyle{ \forall_{x\in \mathbb{R}} f(g(x))=g(f(x)) \wedge f(h(x))=h(f(x))}\) jest identyczność ?
kaszubki
6. Mistrzostwa Rumunii w Matematyce
Problem rozwiązany link .


60. Znów trójkąt Pascala
Znaleźć wszystkie pary liczb naturalnych, \(\displaystyle{ n}\) oraz \(\displaystyle{ k, 2 < k < n}\), takich że \(\displaystyle{ \binom{n}{k-1}, \binom{n}{k}, \binom{n}{k+1}}\) tworzą rosnący ciąg arytmetyczny.
rochaj
[Ciągi] Ciąg arytmetyczny
Ukryta treść:    
Problem rozwiązany przez użytkownika czekoladowy.


61. Mix, Kongruencja z silnią
Dla jakich \(\displaystyle{ n \in\NN}\) jest \(\displaystyle{ \lfloor \frac{1}{3} n \rfloor ! \equiv 0 \ (mod \ n)}\) ? W szczególności scharakteryzować rozmieszczenie tych liczb w zbiorze \(\displaystyle{ \NN}\)
mol_ksiazkowy
mix [MIX][Wielomiany][Teoria liczb] Wielomiany i teoria liczb
Problem rozwiązany połowicznie przez użytkownika Ponewor.


62. Niby proste równanie diofantyczne
Jak rozwiązać w liczbach naturalnych takie równanie?

\(\displaystyle{ x^2 + y^2 = z^2 -1}\)
oszust001
Rozwiązać w liczbach naturalnych
Ukryta treść:    


63. Nierówność z funkcją \(\displaystyle{ S}\)
Udowodnij, że dla dowolnej stałej \(\displaystyle{ k}\) istnieje nieskończenie wiele takich liczb całkowitych \(\displaystyle{ n}\), że \(\displaystyle{ S(2^n+n)+k<S(2^n)}\), gdzie \(\displaystyle{ S(a)}\) to suma cyfr \(\displaystyle{ a}\) w zapisie dziesiętnym.
Swistak
[Teoria liczb] Mocniejsza wersja z 2 etapu

64. Podzbiory kwadratowe
Podzbiór zbioru liczb naturalnych nazywamy kwadratowym jeśli dla każdych dwóch elementów należących do tego podzbioru, ich iloczyn powiększony o 1 jest kwadratem pewnej liczby naturalnej.

Pokazać, że zbiór kwadratowy jest skończony. Znaleźć maksymalną możliwą liczbę elementów zbioru kwadratowego.
matex_06
[Teoria liczb] teoria liczb IRAN
Ukryta treść:    


65. Całkowita suma
Znajdź wszystkie liczby naturalne \(\displaystyle{ a, b}\) takie, że \(\displaystyle{ \frac{a}{b+1}+\frac{b}{a+1}}\) też jest liczbą naturalną.
czekoladowy
[Teoria liczb] Suma ułamków jako liczba całkowita
Ukryta treść:    
Problem rozwiązany połowicznie przez użytkownika arek1357.


66. Twierdzenie Steinhausa
Dla dowolnej liczby naturalnej \(\displaystyle{ n}\) istnieje na płaszczyźnie koło zawierające wewnątrz dokładnie \(\displaystyle{ n}\) punktów kratowych.

Potrzebuję nie tyle dowód tego twierdzenia (chociaż jeżeli nie jest skomplikowany to może być w odpowiedzi), co przykład jego zastosowania w zadaniach...
aga.gmail
twierdzenie Steinhausa
Ukryta treść:    
Problem rozwiązany przez użytkownika mol_ksiazkowy.


67. Przecięcia stycznych
W trójkącie \(\displaystyle{ ABC}\) punkty \(\displaystyle{ E}\) i \(\displaystyle{ F}\) są spodkami wysokości opuszczonych z wierzchołków odpowiednio \(\displaystyle{ B}\) i \(\displaystyle{ C}\). Pokazać, że styczne do okręgu opisanego na trójkącie \(\displaystyle{ AEF}\) w punktach \(\displaystyle{ E}\) i \(\displaystyle{ F}\) przecinają się w punkcie leżącym na odcinku \(\displaystyle{ BC}\).
blebla
Planimetria trójkąt i styczne do okręgu
Problem rozwiązany przez użytkownika Geftus .


68. Własność pewnej funkcji
Niech \(\displaystyle{ S(n)= \sum_{d |n} \tau(d)}\). Czy istnieją takie \(\displaystyle{ n}\), iż \(\displaystyle{ S(n)=n}\) ? Jeśli tak wyznaczyć je wszystkie, przy czym \(\displaystyle{ \tau(n)}\) oznacza ilość dzielników \(\displaystyle{ n}\)
mol_ksiazkowy
Funkcja i równanie
Ukryta treść:    
Problem rozwiązany przez użytkownika marcin_smu .


69. Wielokrotności piątki
Wykazać, że istnieją liczby podzielne przez \(\displaystyle{ 5 ^{1000}}\) nie zawierające w swoim zapisie dziesiętnym ani jednego zera.
maciej.woznica
wykazać, że istnieją liczby...
Problem rozwiązany przez użytkownika marcin_smu .


70. Pokrycie \(\displaystyle{ \NN}\)
Wykazać, że jeżeli mamy dwie dowolnie wybrane liczby rzeczywiste \(\displaystyle{ a,b}\) takie że \(\displaystyle{ a<b}\), to spośród wszystkich przedziałów postaci \(\displaystyle{ (a+2p \pi ,b+2p \pi )}\) gdzie \(\displaystyle{ p \in \mathbb{Z}}\), co najmniej jeden z tych przedziałów zawiera pewną liczbę naturalną.
tatteredspire
przedział do którego należy liczba naturalna

71. Układ, cztery zmienne
Rozwiąż układ równań:

\(\displaystyle{ \begin{cases} abcd= -1 \\ a+b+c+d= 2 \\ a^{2}+ b^{2} + c ^{2}+d ^{2}=8 \\ a ^{3}+b ^{3} +c ^{3} +d ^{3}=5 \end{cases}}\)
Serkuson
Skomplikowany układ równań

72. Własność czworokąta
Punkt \(\displaystyle{ S}\) jest środkiem okręgu wpisanego w pewien czworokąt. Wykazać, że punkt \(\displaystyle{ S}\) jest współliniowy ze środkami przekątnych tego czworokąta.
platynamen
Środek okręgu wpisanego w czworokąt
Ukryta treść:    
Problem rozwiązany przez użytkownika mol_ksiazkowy.


73. Trzy średnie
Średnią kontrharmoniczną liczb dodatnich \(\displaystyle{ x, y}\) nazywamy liczbę \(\displaystyle{ C=\frac{x^2+y^2}{x+y}}\). Wykazać, że \(\displaystyle{ A^2 \geq HC}\), gdzie \(\displaystyle{ A}\) oznacza średnią arytmetyczna, zaś \(\displaystyle{ H}\) średnią harmoniczną liczb \(\displaystyle{ x}\) i \(\displaystyle{ y}\).
mol_ksiazkowy
Zawody Matemat. im M Rejewskiego
Problem rozwiązany przez użytkownika mol_ksiazkowy.


74. Ortocentrum
Na czworokącie \(\displaystyle{ ABCD}\) można opisać okrąg. Niech \(\displaystyle{ P}\) będzie punktem symetrycznym do \(\displaystyle{ C}\) względem \(\displaystyle{ AB}\), a \(\displaystyle{ Q}\) punktem symetrycznym do \(\displaystyle{ C}\) względem \(\displaystyle{ AD}\). Udowodnić, że \(\displaystyle{ PQ}\) przechodzi przez ortocentrum trójkąta \(\displaystyle{ ABD}\).
adriano1992
[Planimetria] Trzy geometrie z okręgami

75. Własność liczb Mersenne'a
Udowodnij, że liczba Mersenne'a postaci \(\displaystyle{ 2^p-1}\) dla \(\displaystyle{ p\in\mathbb{P}}\), \(\displaystyle{ p>2}\) jest pierwsza wtedy i tylko wtedy, gdy zachodzi:

\(\displaystyle{ \sum _{k=0}^{2^p-3} 3^k \equiv 0 \pmod {2^p-1}}\)
JakimPL
[Teoria liczb] Liczby Mersenne'a

76. Hardkorowa trygonometria
Niech \(\displaystyle{ \sin\left({x+y}\right) = 2\sin\left({\frac{{x-y}}{2}}\right)}\) oraz \(\displaystyle{ \sin\left({y+z}\right) = 2\sin\left({\frac{{y-z}}{2}}\right)}\).

Pokaż że \(\displaystyle{ \root 4\of{\frac{1}{2}\sin x\cos z}+\root 4\of{\frac{1}{2}\cos x\sin z}=\root{12}\of{\sin 2y}}\)
darek20
[Trygonometria] Hardkorowa tożsamość trygonometryczna

77. Punkty w kole
Niech \(\displaystyle{ S}\) będzie zbiorem 25 punktów, takim, że w każdym 3-elementowym podzbiorze istnieją dwa punkty, których odległość nie przekracza 1. Udowodnić, że istnieje 13-elementowy podzbiór zbioru \(\displaystyle{ S}\), który można przykryć kołem o promieniu 1.
kordi1221
Zbiór punktów
Problem rozwiązany przez użytkownika mol_ksiazkowy.


78. Okrąg i punkty
Danych jest \(\displaystyle{ 110}\) punktów wewnątrz okręgu jednostkowego. Pokaż, że co najmniej cztery z tych punktów leżą wewnątrz okręgu o promieniu \(\displaystyle{ \frac{1}{8}.}\)
darek20
[Kombinatoryka] punkty w okregu

79. Mix, ogórki III; Współstyczne koła
Na płaszczyźnie umieszczone są koła o rozłącznych wnętrzach, i każde koło jest styczne do co najmniej sześciu spośród pozostałych kół. Udowodnić, że kół tych jest nieskończenie wiele. Czy istnieje na płaszczyźnie skończona ilość kół o rozłącznych wnętrzach , z których każde jest styczne do pewnych pięciu spośród pozostałych kół ?
mol_ksiazkowy
[MIX] Zestaw zadań na sezon ogórkowy III

80. Zbiory i elementy
Dany zbiór \(\displaystyle{ X}\)
\(\displaystyle{ A _{1} , A _{2} , A_{3} ,..., A_{1066}}\) - podzbiory \(\displaystyle{ X}\)
dla każdego \(\displaystyle{ i \left|A _{i}\right| > \frac{1}{2} |X|}\)

Pokazać, że istnieją \(\displaystyle{ x _{1} ,...,x _{10} \in X}\) takie, że dla każdego\(\displaystyle{ i \in {1,...,1066}}\) istnieje \(\displaystyle{ j \in {1,...,10} \ x _{j} \in A _{i}}\)
Nominalista
Zasada szufladkowa Dirichleta

81. Ilość ciągów
Ile jest ciągów o długości \(\displaystyle{ 2n}\) takich, że każda liczba \(\displaystyle{ i\in\left\{1,2, ... ,n \right\}}\) występuje dokładnie dwa razy oraz każde sąsiednie dwa wyrazy są różne ?
pavel3643
Ile jest ciągów długości 2n...
Problem rozwiązany przez użytkownika mol_ksiazkowy.


82. Jednokolorowe wierzchołki
Wszystkie krawędzie sześcianu oraz jego przekątne i przekątne jego ścian pomalowano na biało lub czarno (każdy z wymienionych odcinków na jeden kolor). Udowodnij, że istnieją dwa różne czworokąty o wierzchołkach w wierzchołkach danego sześcianu i bokach jednego koloru.

[MIX] Zadania treningowe przed drugim etapem OM - 3 seria.

83. Uczeni na konferencji
W międzynarodowej konferencji naukowej wzięło udział \(\displaystyle{ 1998}\) uczonych. Wśród każdych trzech naukowców co najmniej dwóch włada tym samym językiem, a każdy z uczestników konferencji zna co najwyżej pięć języków. Udowodnij, że co najmniej \(\displaystyle{ 200}\) uczonych zna ten sam język.

[MIX] Zadania treningowe przed drugim etapem OM - 1 seria.

84. Kliki w grafach
Wiemy, że zachodzi następujące twierdzenie :

"każdy graf nieskierowany o sześciu wierzchołkach i przynajmniej dziesięciu krawędziach zawiera klikę trzyelementową"

Korzystając z tego faktu udowodnij, że każdy graf nieskierowany, który ma osiem wierzchołków i nie mniej niż \(\displaystyle{ 17}\) krawędzi, zawiera klikę trzyelementową.
rzexnik
Trudne(?) grafy

85. Własność liczby \(\displaystyle{ 2821}\)
Pokazać, że 2821 jest liczbą Carmichaela.
gelo21
liczba Carmichaela
Ukryta treść:    
Problem rozwiązany przez użytkownika mol_ksiazkowy i Afish .


86. Mix 31; Liczby trójkątne
Wykazać, że istnieje nieskończenie wiele liczb parzystych, będących różnicami dwóch liczb trójkątnych tylko na jeden sposób.
mol_ksiazkowy
[MIX] Mix matematyczny (31)
Ukryta treść:    
Problem rozwiązany link .


87. W trójkącie Pascala
dla jakich n calkowitych wiekszyc od \(\displaystyle{ 1}\) liczby \(\displaystyle{ {n \choose 1}{n \choose 2},...,{n \choose n-1}}\) maja wspólny dzielnik wiekszy od \(\displaystyle{ 1}\).

Doszedłem do tego że jak \(\displaystyle{ n}\) jest pierwsze to wszystko działa, jak \(\displaystyle{ n}\) jest potęgą liczby pierwszej to wydaje mi się (dowodu nie umiem) ze też jest ok.; jak jest złożona to nie działa (tez nie umiem udowodnić)
qsiarz
[Kombinatoryka][Teoria liczb] Symbol Newtona, podzielnosc.
Ukryta treść:    
Problem rozwiązany link .


88. Zbiór z nwd
Niech zbiór \(\displaystyle{ A}\) zawiera \(\displaystyle{ n}\) liczb naturalnych. Pokaż że zbiór postaci \(\displaystyle{ \left\{\frac{ab}{nwd (a, b)^{2}}: a, b\in A\right\}}\) zawiera co najmniej n elementów.
darek20
[Teoria liczb] zbiór z nwd

89. Podzielność i nieparzystość
Niech \(\displaystyle{ m,n}\) naturalne Pokazać że jeśli \(\displaystyle{ \frac{(m+3)^n+1}{3m}}\) całkowite to \(\displaystyle{ \frac{(m+3)^n+1}{3m}}\) jest nieparzyste.
marek12
[Teoria liczb] nieparzysta liczba



90. Rozkład na sumę
Niech \(\displaystyle{ k>2}\) naturalne. Liczby \(\displaystyle{ k}\)-kątne to elementy ciągu \(\displaystyle{ a_n = k\cdot\frac{n^2-n}{2}-n(n-2)}\)

Twierdzenie Cauchy'ego (znalezione w Teorii liczb Narkiewicza)

Dla dowolnego \(\displaystyle{ n>2}\), każda liczba naturalna da się zapisać w postaci sumy co najwyżej \(\displaystyle{ n}\) liczb \(\displaystyle{ n}\)-kątnych.
rnd01001
Liczby k-kątne

91. Lemat o liczbach pierwszych
Jak udowodnić takie twierdzenie:

Niech \(\displaystyle{ p>2}\) liczba pierwsza oraz \(\displaystyle{ k\in\{1,2,\dots,2p+2\}\setminus\{p,2p\}}\). Niech \(\displaystyle{ N=2kp+1}\). Następujące warunki równoważne są:

(1) \(\displaystyle{ N}\) jest liczbą pierwszą
(2) istnieje \(\displaystyle{ a\in\{2,3,\dots,N-1\}}\) takie że \(\displaystyle{ N\mid a^{kp}+1}\) oraz \(\displaystyle{ \gcd(a^k+1,N)=1}\) ?
rnd01001
uogólnione liczby Sophie Germain

92. Mix; kongruencja
Niech \(\displaystyle{ p>2}\) bedzie liczbą pierwsza a \(\displaystyle{ k}\) liczbą naturalną . Pokaż że

\(\displaystyle{ \sum_{j=0}^{k}\binom{k(p-1)}{j(p-1)}\equiv 2+p(1-k) (\mod p^{2})}\)
rochaj
[MIX] MIX na sezon bez szkoły

93. Max mix; Iloczyn
Niech \(\displaystyle{ x, a, b}\) to będą liczby naturalne , takie iż \(\displaystyle{ x^{a+b}=a^b b}\). Wykaż, ze \(\displaystyle{ a=x}\) i \(\displaystyle{ b=x^x.}\)
mol_ksiazkowy
[MIX] max mix

94. Nierówność dla ciągu
Niech \(\displaystyle{ \{a_i\}_{0 \le i \le n}}\) będzie niemalejącym i nieujemnym ciągiem. Niech \(\displaystyle{ b_j= \sum_{i=j}^{n} {i \choose j} a_i}\). Pokaż że \(\displaystyle{ b_j^2 \ge b_{j-1}b_{j+1}}\).
darek20
[Ciągi][Nierówności] wykazanie nierówności

95. Iloczyn i suma
Wykaż że

\(\displaystyle{ (1+x)(1+x^3)(1+x^5)... = 1 + \sum_{k=1}^{\infty} \frac{ x^{k^2} }{ (1-x^2)(1-x^4)... (1-x^{2k})}.}\)
robin5hood
[Równania] wykazanie równości
Ukryta treść:    


96. Wielomian i podzielność
Niech \(\displaystyle{ k\in \NN^*}\) i wielomian \(\displaystyle{ P(x)}\) taki że \(\displaystyle{ P(x)\in \ZZ}\) dla \(\displaystyle{ x\in \ZZ}\).

\(\displaystyle{ xP(x)-P(x-1)=x^k}\).

Pokaż że \(\displaystyle{ 3|k-2}\).
robin5hood
[Wielomiany] wielomian i podzielność
Ukryta treść:    


97. Kostki
Mamy duży zapas kostek sześciennych przezroczystych i zielonych, które są jednakowej wielkości. Z kostek takich budujemy bryły sześcienne, a kostki zielone umieszczamy w takich miejscach i w takiej liczbie, aby spełniony był następujący warunek: Jeżeli przez środek każdej kostki zielonej przeprowadzimy trzy proste równoległe do krawędzi zbudowanej bryły, to każda kostka użyta do budowy tej bryły zostanie przebita co najmniej jedną z tych prostych. Jaką najmniejszą liczbę zielonych kostek trzeba użyć do budowy bryły sześciennej złożonej ze 125 kostek ,aby spełniony był wyżej podany warunek ?
pitgot
Optymalizacja przy podanym warunku

98. Maksimum sum
Dla każdej pary liczb całkowitych \(\displaystyle{ (x,y)}\), spełniających równanie \(\displaystyle{ (x^2+y^2)(x-2y+15)=2xy}\), oblicz sumę \(\displaystyle{ x\!+\!y}\). W odpowiedzi podaj największą możliwą sumę.
sir George
Open Mathematical Olympiad of the Belarusian-Russian Univ
Problem rozwiązany przez użytkownika Ponewor.


99. Nierówność Shapiro
Niech \(\displaystyle{ n\in \mathbb{N}_{3} = \{3, 4, 5, \ldots\}}\) oraz \(\displaystyle{ 0<a_{1}\leqslant a_{2}\leqslant \ldots \leqslant a_{n}}\). Wykaż, że:

\(\displaystyle{ \frac{a_{1}}{a_{2} + a_{3}} + \ldots + \frac{a_{n-2}}{a_{n-1} + a_{n}} + \frac{a_{n - 1}}{a_{n} + a_{1}} + \frac{a_{n}}{a_{1} + a_{2}}\geqslant \frac{n}{2}}\).
max
[Nierówności] Nierówność cykliczna
Ukryta treść:    


100. Własność liczb pierwszych
Czy istnieją \(\displaystyle{ n\in \mathbb{N}_{3} = \{3, 4, 5, \ldots\}}\) i \(\displaystyle{ p_{1}, \ldots, p_{n}}\) parami różne liczby pierwsze takie, że \(\displaystyle{ p_{i}\not\equiv 1\pmod{p_{j}}}\) dla \(\displaystyle{ i\neq j}\), że dla każdego \(\displaystyle{ l \in \{1, \ldots, n\}}\) istnieją \(\displaystyle{ k\in \{1, \ldots, n\}}\) i parami różne \(\displaystyle{ j_{1},\ldots j_{k}\in \{1,\ldots, n\}}\), że zachodzi:

\(\displaystyle{ p_{j_{1}}\cdot \ldots \cdot p_{j_{k}} \equiv 1 \pmod{p_{l}}}\) ?

(z odpowiedzi negatywnej wynikałoby istnienie ciekawego rozwiązania pewnego innego problemu).
max
Istnienie liczb pierwszych spełniających układ kongruencji
Problem rozwiązany przez użytkownika Ponewor.


101. Szacowanie stopni
Niech \(\displaystyle{ m,n\in \mathbb{N}, \ m,n\ge 2,}\) niech \(\displaystyle{ f,g}\) będą wielomianami (o współczynnikach rzeczywistych, albo ogólniej - zespolonych), przy czym \(\displaystyle{ f^{m} - g^{n}\neq 0.}\)

Wykaż, że \(\displaystyle{ \deg (f^{m} - g^{n})\ge \frac{mn - m - n}{n}\deg f + 1.}\)
max
[Wielomiany] Oszacowanie stopnia różnicy potęg wielomianów
Awatar użytkownika
mol_ksiazkowy
Użytkownik
Użytkownik
Posty: 11263
Rejestracja: 9 maja 2006, o 12:35
Płeć: Mężczyzna
Lokalizacja: Kraków
Podziękował: 3140 razy
Pomógł: 747 razy

[MIX] Nierozwiązane problemy

Post autor: mol_ksiazkowy »

N \(\displaystyle{ \sqrt{-1}}\) ero \(\displaystyle{ z}\) w \(\displaystyle{ \sqrt{-1}}\) ą \(\displaystyle{ z}\) ane Problemy 5

1. Mix zadania różne VI
Lemat o grupie jednorodnej; Udowodnić iż w ośmioosobowej grupie osób istnieją rozłączne grupy jednorodne trzyosobowe
Grupa jednorodna to taka, w której każdy zna każdego lub nikt nie zna nikogo
(tez twierdzenia Turána)
mol_ksiazkowy
[MIX] UW kontratakuje (Mix zadań)

2. Mix zadania różne I, Zadanie 30
Niech \(\displaystyle{ \alpha}\) będzie rozwiązaniem równania \(\displaystyle{ x^5+x-1=0}\). Wyznaczyć równanie, którego rozwiązaniem jest \(\displaystyle{ 1+ \alpha^4}\).
mol_ksiazkowy
[MIX] Zadania różne
Problem rozwiązany przez użytkownika Hydra147.
3. Mix zadania różne VII
Czy istnieją \(\displaystyle{ a, b >1}\) takie, że
\(\displaystyle{ \begin{cases} a^4 \ \equiv 1 \ \left( \bmod b^2 \right) \\ b^4 \ \equiv 1 \ \left( \bmod a^2 \right) \end{cases}}\)
?
mol_ksiazkowy
[MIX] Zadania różne VII

4. Mix zadania różne IV
Wewnątrz kwadratu jest zbiór odcinków o końcach na obwodzie kwadratu. Suma długości tych odcinków jest równa 3. Wykazać, że gdy \(\displaystyle{ 8r<1}\) to w kwadracie tym istnieje koło o promieniu \(\displaystyle{ r}\) rozłączne z każdym z tych odcinków (tj. nie przecinające żadnego z nich).
mol_ksiazkowy
[MIX] Zadania różne IV

5. Mix zadania różne VIII, zadanie 5 konstrukcja
Mając dany okrąg i trzy niewspółliniowe punkty w jego wnętrzu skonstruować trójkąt wpisany w ten okrąg i taki, że na każdym jego boku jest jeden z tych punktów.
mol_ksiazkowy
[MIX] Zadania różne VIII

6. Mix Zadania z setką ; Zadanie 14
100 zawodników rozgrywa turniej szachowy systemem każdy z każdym. Po 32 rundach turniej został przerwany. Wykazać, że co najmniej 4 pary graczy uzyskały ten sam wynik.
mol_ksiazkowy
[MIX] Zadania z setką

7. Mix zadania różne X; Zadanie 11
W okrąg o obwodzie \(\displaystyle{ 24}\) wpisany jest trójkąt równoboczny oraz kwadrat i nie maja one wspólnego wierzchołka. Wykazać, że co najmniej jeden z siedmiu łuków, na które te wierzchołki podzieliły okrąg ma długość nie większą niż \(\displaystyle{ 1}\).
mol_ksiazkowy
[MIX] Zadania różne X

8. Suplement KMDO, Zadanie 162
Ciąg \(\displaystyle{ (x_n)}\) określony jest następująco
\(\displaystyle{ x_1 =2 \quad x_{n+1} =\left[ \frac{3}{2} x_n \right] \quad \text{dla } \; n=1,2,3,\ldots}\)
Udowodnij, że
\(\displaystyle{ 1^\circ}\) w ciągu \(\displaystyle{ (x_n)}\) występuje nieskończenie wiele liczb parzystych i nieskończenie wiele liczb nieparzystych.
\(\displaystyle{ 2^\circ}\) ciąg \(\displaystyle{ \left( (-1)^{x_n} \right)}\) nie jest okresowy.
frej
[MIX] Suplement KMDO

9. Nierówność na płaszczyźnie
Dla pięciu punktów \(\displaystyle{ A,B,P,Q,R}\) na płaszczyźnie pokaż że
\(\displaystyle{ AB+PQ+QR+RP \leq AP+AQ+AR+BP+BQ+BR.}\)
darek20
[Nierówności][Planimetria] pięć punktów

10. Mix IX, Zadanie o czworobloku
Czworoblok to figura podzielona na cztery przystające wielokąty, tak iż część wspólna każdych dwóch z nich zawiera odcinek. Czy istnieją czworobloki wypukłe ?
Ukryta treść:    
mol_ksiazkowy

11. Trójkąt i liczba
Czy dla każdej liczby wymiernej \(\displaystyle{ q}\) istnieje trójkąt, którego długości boków są liczbami wymiernymi oraz jego pole jest równe \(\displaystyle{ q}\) ? Odpowiedz uzasadnij.
rochaj
[Planimetria] trójkat o polu wymiernym

12. Punkty w kwadracie
W kwadracie o boku \(\displaystyle{ 1}\) znajduje się \(\displaystyle{ m^2}\) punktów, tak że każde trzy nie leżą na jednej prostej. Pokaż ze istnieje trójkąt, którego wierzchołkami są trzy spośród tych punktów oraz pole tego trojka jest nie większe niż \(\displaystyle{ \frac{1}{2(m-1)^2}.}\)
alfred0
[Planimetria] punkty w kwadracie

13. Nierówność w trójkącie
Niech dany jest \(\displaystyle{ \Delta ABC}\) o wysokościach \(\displaystyle{ h_{a},h_{b},h_{c}}\) gdzie \(\displaystyle{ AB=c,BC=a,AC=b}\)
Pokaż że dla dowolnego punktu \(\displaystyle{ P}\) wewnatrz tego trójkata mamy
\(\displaystyle{ \sqrt{PA+PB}+\sqrt{PB+PC}+\sqrt{PA+PC}\ge 2\sqrt{h_{a}+h_{b}+h_{c}}}\)
rochaj
[Nierówności] nierówność w trójkacie

14. Wartość funkcji
Niech \(\displaystyle{ f(x)}\) będzie funkcją ciągła na \(\displaystyle{ \left [ 0;1 \right ]}\) i taka że:
1) \(\displaystyle{ f(0)=0, f(1)=1}\)
2) \(\displaystyle{ 5f\left (\frac{3x+y}{4}\right) = 4f(x)+f(y)}\) dla każdego \(\displaystyle{ x,y \in \left [ 0;1 \right ]}\) oraz \(\displaystyle{ x\geq y}\)
Oblicz \(\displaystyle{ f\left( \frac{27}{55}\right).}\)
rochaj
[Funkcje][Równania funkcyjne] wartość funkcji

15. Graf
Mamy graf. I jest on nieskierowany, spójny. Ma \(\displaystyle{ 100}\) wierzchołków. Ma taką własność: każdy podgraf ma wierzchołek (choć jeden) o takiej własności, że jego stopień to jest nie większy niż \(\displaystyle{ 10}\).
Udowodnić, że liczba wierzchołków stopnia co najmniej \(\displaystyle{ 30}\) jest mniejsza niż \(\displaystyle{ 66}\).
matinf
grafe o pewnej własnosci.

16. Gra
Rozważmy grę, która polega na obstawianiu orła lub reszki na ustaloną przez nas kwotę, przyjmijmy \(\displaystyle{ k}\). Jeżeli obstawimy stronę, która wypadnie, do naszego kapitału wpływa \(\displaystyle{ k}\), natomiast jeżeli przegramy, musimy oddać \(\displaystyle{ k}\). Obie strony wypadają z równym sobie prawdopodobieństwem \(\displaystyle{ \frac{1}{2}}\). Gracz gra strategią następującą:
- w pierwszym kroku stawia \(\displaystyle{ 1}\) na orła (zauważmy, że obstawiany typ jest bez znaczenia),
- jeżeli przegra, podwaja stawkę; jeżeli jest to niemożliwe - gra va banque,
- jeżeli wygra, ponawia krok pierwszy, stawiając \(\displaystyle{ 1}\).

Jakie jest prawdopodobieństwo, że, startując z kapitałem \(\displaystyle{ 2^N}\), przegra cały swój majątek, jeżeli gra kończy się nie później niż po \(\displaystyle{ n}\) obstawieniach? Rozważyć nieskończoną odmianę tej gry iterowanej.
JakimPL
[Prawdopodobieństwo] Nieskończona gra w obstawianie

17. Równanie funkcyjne
Znaleźć takie funkcje \(\displaystyle{ f}\), że dla każdego \(\displaystyle{ x,y \in \RR}\) zachodzi \(\displaystyle{ f(f(x)+y)+f(f(y)+x) = x+y.}\)
robson161
[Równania funkcyjne] Autorskie równanie funkcyjne

18. Maksimum iloczynu
Niech \(\displaystyle{ x,y\in \RR}\) tak że \(\displaystyle{ x+y=3(\sqrt{x-2}+\sqrt{y+1}-1).}\) Znaleźć maksimum \(\displaystyle{ xy}\).
rochaj
maksimum iloczynu

19. Funkcja i nierówność
Niech \(\displaystyle{ f(x)=\frac{e^{\frac{1}{x}}-1}{e^{\frac{1}{x+1}}-1}}\). Pokaż ze dla każdego \(\displaystyle{ x\in (0,\infty)}\) mamy
\(\displaystyle{ f(x)>\sqrt{\frac{x+1}{x}}}\) oraz \(\displaystyle{ f(x)<\sqrt{\frac{x+1}{x-1}}}\).
rochaj
[Analiza] Wykazanie nierównośći z funkcja wykładniczą

20. Równanie modulo
Znaleźć wszystkie liczby całkowite \(\displaystyle{ x}\) takie, że :
\(\displaystyle{ x \equiv 4 \ (\bmod 7)}\) i \(\displaystyle{ 62x \equiv \ 102 \ (\bmod 162).}\)
Hodor
Równanie modulo
Problem rozwiązany połowicznie przez użytkownika Hodor.
21. Sumy kwadratów kolejnych liczb
Czy i jakie i dla jakich \(\displaystyle{ n}\) równanie :
\(\displaystyle{ a^2+ (a+1)^2+…+ (a+n)^2 = b^2}\)
ma rozwiązania w zbiorze \(\displaystyle{ \NN}\) ?
mol_ksiazkowy
Sumy kwadratów kolejnych liczb
Problem rozwiązany połowicznie przez użytkownika mol_ksiazkowy.
22. Ile cyfr ?
\(\displaystyle{ \huge \left(2^{43,112,609}-1\right)^{13\cdot 2^{43,112,609}-14}}\)
ile cyfr ma liczba mówiąca o tym ile cyfr ma liczba określająca liczbę cyfr tej liczyby
Xitami
Ile cyfr

23. Suma z Mixa
Obliczyć \(\displaystyle{ \sum_{n}^{k=1} \frac{1}{(k+1) \sqrt{k} + k \sqrt{k+1} }}\)
laurelandilas
[MIX] Wymierność i wartość bezwzględna

24. Hipoteza
Proszę o pomoc w zweryfikowaniu następującej hipotezy (albo jakiekolwiek informacje z nią związane, jeśli jest znana):
Dla każdej liczby pierwszej \(\displaystyle{ p}\) i dla każdego całkowitego dodatniego \(\displaystyle{ n}\) zachodzi:
\(\displaystyle{ p \nmid 2^{2^{n}}+1 \vee p^{2} \nmid 2^{2^{n}}+1}\)
Ponewor
Hipoteza dotycząca liczb Fermata

25. Nieistniejący ciąg
Pokaż że istnieje ciąg \(\displaystyle{ a_n\in\mathbb{N}}\) taki że \(\displaystyle{ a_n \le n!e\le a _n + \frac{2}{n+1}}\)
rochaj
istnienie ciagu

26. Granica z arcusem
Oblicz
\(\displaystyle{ \lim_{x\rightarrow 2}\arctan\left [\frac{96}{11}\left( \frac{\sqrt[4]{x+14}-\sqrt[3]{10-x}}{x-2}\right)\right ].}\)
rochaj
granica funkcji

27. Różniaste z podzielności; Mix
Wykazać, że \(\displaystyle{ i_{1977}}\) ma co najwyżej 364 dzielniki naturalne; przy czym \(\displaystyle{ i_n= \underbrace{1 \cdots 1}_{n}}\)
mol_ksiazkowy
[MIX] Różniaste z podzielności

28. Nierozkładalny wielomian
Pokaż że wielomian \(\displaystyle{ x^{2p}+px^n-1}\) jest nierozkładalny, gdzie \(\displaystyle{ p}\) to liczba pierwsza.
rochaj
wielomian nierozkładalny

29. Równanie 4 stopnia
Znaleźć wszystkie \(\displaystyle{ x, y, z \in \ZZ^{+}}\) takie, że \(\displaystyle{ x^4-4x^2y^2-4y^4=z^2.}\)
darek20
[Teoria liczb] Równanie w liczbach naturalnych

30. Mix matematyczny 33; Problem 3
Liczby ze zbioru \(\displaystyle{ \{ 1, ..., 4k \}}\) rozmieszczono w dowolny sposób przypisujac je różnym \(\displaystyle{ 4k}\) punktom na okręgu. Udowodnić, że istnieje \(\displaystyle{ 2k}\) rozłącznych cięciw i takich, że różnica liczb na obu końcach każdej z tych cięciw jest nie większa niż \(\displaystyle{ 3k-1}\)
mol_ksiazkowy
[MIX] Mix matematyczny 33

31. Wartość funkcji
Niech \(\displaystyle{ f: \mathbb{R} \to \mathbb{R}}\) będzie funkcją mającą własność Darboux na \(\displaystyle{ \mathbb{R}}\). Pokaż że jeśli istnieje \(\displaystyle{ m>0}\) takie że \(\displaystyle{ |f(x)-f(y)| \ge m|x-y|,}\) dla \(\displaystyle{ x,y \in \mathbb{R}}\) to f jest bijekcją.
darek20
własność Darboux i bijekcja

32. Niewymierność sumy logarytmów
wykazać że \(\displaystyle{ \log_3 5 + \log_2 5}\) jest liczbą niewymierną.
ms7
Niewymierność sumy logarytmów o różnych podstawach

33. Mix Zadania różne III
Udowodnić, że dwie parabole, których osie symetrii są prostopadłe przecinają się w czterech punktach współokręgowych
mol_ksiazkowy
[MIX] Zadania różne III
Problem rozwiązany przez użytkownika timon92.
34. Mix Zadania różne VI, Zadanie 26
Wykazać że iloczyn wszystkich \(\displaystyle{ 2^{100}}\) liczb \(\displaystyle{ \pm \sqrt{1} \pm \sqrt{2} \pm ... \pm \sqrt{99} \pm \sqrt{100}}\)
jest kwadratem liczby całkowitej.
mol_ksiazkowy
[MIX] Zadania różne VI

35. Pół kwadrat
Czy jeśli \(\displaystyle{ p}\) jest liczbą pierwszą \(\displaystyle{ p=3k+1}\), to istnieją \(\displaystyle{ a, b \in \NN}\)\(\displaystyle{ p=a^2+ab+b^2}\) tj. \(\displaystyle{ p}\) jest pół kwadratem sumy. Czy taki rozkład jest jednoznaczny ?
mol_ksiazkowy
Pół kwadrat

36. Symetryczny układ
Przeanalizować ten układ:
\(\displaystyle{ \begin{cases} x^k+ y = a\\ y^k+ x =a\end{cases}}\)
gdy \(\displaystyle{ k \in \NN}\) a \(\displaystyle{ a \in \RR}\)
mol_ksiazkowy
Symetryczny układ

37. Wyznacznik, trzy zmienne
Udowodnić, że jeśli:
\(\displaystyle{ A= \begin{bmatrix} 1+x^2-y^2-z^2&2(xy+z)&2(zx-y)\\2(xy-z) &1+y^2-x^2-z^2&2(yz+x)\\2(zx+y)&2(yz-x)&1+z^2-x^2-y^2\end{bmatrix}}\)
to wyznacznik macierzy \(\displaystyle{ A}\) to \(\displaystyle{ (1+x^2+y^2+z^2)^3.}\)
mol_ksiazkowy
wyznacznik, trzy zmienne

38. Suma z cechą
Ile to jest \(\displaystyle{ \sum_{k=1}^n \lfloor (\frac{k}{3})^3 \rfloor}\) ?
mol_ksiazkowy
Suma z cechą
Problem rozwiązany połowicznie przez użytkownika liu.
39. Mix Zadania różne X; Problem 25
Udowodnić, że \(\displaystyle{ \frac{1}{\sin^2(\frac{\pi}{2n})}+ \frac{1}{\sin^2(\frac{2\pi}{2n})}+ .... + \frac{1}{\sin^2(\frac{(n-1)\pi}{2n})} =\frac{2}{3}(n-1)(n+1).}\)
mol_ksiazkowy
[MIX] Zadania różne X
Problem rozwiązany przez użytkownika mol_ksiazkowy.
40. Iloczyn pierwiastków
Udowodnij, że jeśli liczby \(\displaystyle{ a}\) i \(\displaystyle{ b}\) są pierwiastkami wielomianu \(\displaystyle{ x ^{4} + x^{3}-1}\), to liczba \(\displaystyle{ ab}\) jest pierwiastkiem wielomianu \(\displaystyle{ x ^{6}+ x^{4} +x ^{3}-x ^{2} -1}\).
Ukryta treść:    
satre
[Wielomiany] Wielomian z "Zadania z olimpiad matematycznych"

41. Dziwny wyznacznik
Ile jest równy wyznacznik macierzy, w której:
pierwszy rząd to \(\displaystyle{ (a_1, a_2, a_3,...., a_n)}\)
\(\displaystyle{ i}\) ty rząd to \(\displaystyle{ (a_i, a_i, a_i ,...., a_{i+1}, ...., a_n)}\)
\(\displaystyle{ n}\) ty rząd to \(\displaystyle{ (a_n, a_n, a_n, ...., a_n)}\)
?
mol_ksiazkowy
Dziwny wyznacznik

42. Mix zadań, Problem 5
Niech \(\displaystyle{ n}\) będzie liczbą naturalną. Wykaż, że istnieje nieskończenie wiele liczb naturalnych \(\displaystyle{ \left( a,b,c,d,e\right)}\) takich, że
\(\displaystyle{ n = \frac{a^{2}+b^{2}+c^{2}+d^{2}+e^{2}}{abcde+1}.}\)
Zahion
[Teoria liczb] Mix zadań.

43. Nierówność
Pokaż: \(\displaystyle{ -\sqrt{2}\log(\cos x)\leq\sqrt{x\tg x-\sin^{2}x}}\) dla \(\displaystyle{ x\in\left [ 0,\frac{\pi}{2}\right)}\)
darek20
[Nierówności] wykazanie nierówności r-k różniczkowy

44. Mix 11, Zadanie 29
Dane są liczby naturalne \(\displaystyle{ a, b >1}\) względnie pierwsze i takie, że \(\displaystyle{ \frac{a^2}{2ab^2 -b^3 +1}}\) jest liczbą całkowitą. Udowodnić, że \(\displaystyle{ a=7}\) i \(\displaystyle{ b=2}\)
mol_ksiazkowy
[MIX] Zadania różne XI

45. Istnienie liczby naturalnej
Czy dla każdej liczby niewymiernej \(\displaystyle{ x}\) istnieje liczba naturalna \(\displaystyle{ n \ge 1}\) taka, że iloczyn \(\displaystyle{ nx}\) ma w rozwinięciu dziesiętnym nieskończenie wiele zer lub nieskończenie wiele dziewiątek?
scoopler
[Algebra] Istnienie liczby naturalnej

46. Mix Zadania różne XII, kwadraty
Udowodnić, że nie istnieją liczby naturalne \(\displaystyle{ x, y}\) takie że \(\displaystyle{ x^2+x y +y^2}\) oraz \(\displaystyle{ x^2 - x y +y^2}\) są kwadratami liczb całkowitych.
W.S.
mol_ksiazkowy
[MIX] Zadania różne XII

47. Problem szklanych kul
Szklana kula zrzucona z jakiegoś piętra \(\displaystyle{ n}\) piętrowego wieżowca może się rozbić. Zależy to nie od kuli, lecz od wysokości. Mamy \(\displaystyle{ k}\) takich kul. Należy ustalić minimalna ilość prób, które trzeba wykonać, zrzucając kule z pięter, aby zawsze wykryć numer najwyższego pietra, z którego zrzucona kula nie rozbije się.
Niech \(\displaystyle{ f(n, k)=m}\) będzie szukaną ilością prób. Obliczyć \(\displaystyle{ f(206, 4)}\)
mol_ksiazkowy
Problem szklanych kul

48. Mix Zadania różne XIII; Problem 17
Dla jakich liczb całkowitych \(\displaystyle{ m, n}\) liczby \(\displaystyle{ 3^m + 1}\) i \(\displaystyle{ 3^n + 1}\) są podzielne przez \(\displaystyle{ mn}\) ?
mol_ksiazkowy
[MIX] Zadania różne XIII

49. NWD wielomianów
Obliczyć NWD dwóch wielomianów oto one:
\(\displaystyle{ x^3+x^2-x-1}\) i \(\displaystyle{ 3x^2+2x-1}\)
lightinside
NWD dwóch wielomianów używając algorytmu Euklidesa

50. Jednoznaczność działania
Czy jeśli \(\displaystyle{ (\RR, *)}\) jest zbiorem z działaniem takim, że \(\displaystyle{ ( a*b) *c = a+b+c}\) dla \(\displaystyle{ a, b, c \in \RR}\) to \(\displaystyle{ a*b = a+b}\) dla \(\displaystyle{ a, b \in \RR}\) ?
O ile nie to wskazać inne (niż dodawanie) takie działanie
mol_ksiazkowy
Jednoznaczność działania

51. Nierówność ze Zwardonia
Dane są takie liczby rzeczywiste \(\displaystyle{ x_1, x_2, ..., x_n, y_1, ..., y_n}\) spełniające warunek \(\displaystyle{ x_1 ^2+ x_2 ^2+...+x_n^2=y_1^2+...+y_n^2=1}\) oraz \(\displaystyle{ x_1y_1+...+x_ny_n=0}\). Udowodnij, że \(\displaystyle{ (x_1+...+x_n)^2+(y_1+...+y_n)^2 \le n}\)
Swistak
[Nierówności] zawody indywidualne, Zwadroń 2011

52. Tożsamość Li -Żen-szua
Znaleźć dowód kombinatoryczny tożsamości \(\displaystyle{ \sum_{j=0}^{k} {k \choose j}^2 {n+2k-j \choose 2k}= {n+k \choose k}^2}\)
Ukryta treść:    
mol_ksiazkowy
Tożsamość Li-Żen-Szua

53. Szczególny trójkąt
Czy istnieje taki trójkąt o bokach całkowitej długości, że jedna z wysokości ma długość równą długości boku, na który została opuszczona?
KPR
[Teoria liczb] Teoria liczb dla koksów

54. Siedem fajnych Równań funkcyjnych
Niech \(\displaystyle{ f: \RR^{+} \to \left\langle -1, 1 \right\rangle}\)
\(\displaystyle{ f (x+3) - f(x+2) = 3 \lfloor x + \lfloor x \rfloor -2 \lfloor x+1 \rfloor \rfloor}\) dla \(\displaystyle{ x \in \RR}\). Udowodnić, że \(\displaystyle{ f}\) jest okresowa.
mol_ksiazkowy
[MIX] 7 fajnych równań funkcyjnych

55. Tożsamość z sinusem i cosinusem
Pokaż że jeśli \(\displaystyle{ \dfrac{\cos x}{\cos y}+\dfrac{\sin x}{\sin y}=-1}\), to \(\displaystyle{ \dfrac{\cos^3 y}{\cos x}+\dfrac{\sin^3 y}{\sin x}=1}\).
alfred0
[Trygonometria] tożsamość trygonometryczna

56. Obroty na łóżku
Zając kładzie się spać na górnym piętrze łóżka piętrowego. Jest ono ustawione w ten sposób, że z jednej strony jest ściana, a z drugiej nie ma żadnej barierki, więc można spaść. Świstak widząc to zaczyna się zastanawiać jakie jest prawdopodobieństwo, że jego kolega spadnie. Pomóż świstakowi!
Wiadomo, że średnio człowiek w ciągu nocy obraca się \(\displaystyle{ n}\) razy. Jeżeli w pewnym momencie \(\displaystyle{ L-P=k}\), to Zając spadnie, gdzie \(\displaystyle{ L}\) to dotychczasowa liczba obrotów w lewo, a \(\displaystyle{ P}\) w prawo.
kaszubki
[Prawdopodobieństwo] Obroty na łóżku

57. Mix Teoria liczb; łatwe i trudne;
Dla jakich \(\displaystyle{ a}\) i \(\displaystyle{ b}\): \(\displaystyle{ b^2}\) jest podzielne przez \(\displaystyle{ a+1}\) i \(\displaystyle{ a^2}\) jest podzielne przez \(\displaystyle{ b+1}\) ?
mol_ksiazkowy
Ukryta treść:    
[MIX] Teoria liczb, łatwe i trudne

58. Suma tangensow
Pokaż że \(\displaystyle{ \tg1^{\circ}+\tg7^{\circ}+...+\tg175^{\circ}=-30\sqrt3}\), gdzie \(\displaystyle{ 1^{\circ},7^{\circ},...,175^{\circ}}\) tworzą ciag arytmetyczny.
rochaj
[Trygonometria][Ciągi] suma tangensów

59. Policz ciągi
Ile jest ciągów (o wyrazach \(\displaystyle{ 1,2..,m}\)) o długości \(\displaystyle{ n}\) takich, że każda z liczb \(\displaystyle{ 1,2...m}\) wystąpiła w nim przynajmniej raz.
kjnm
Policz ciągi

60. Sumy cyfr
Udowodnij, że dla każdej liczby pierwszej \(\displaystyle{ p>11}\) i dla każdego naturalnego \(\displaystyle{ k}\) istnieje taka \(\displaystyle{ m}\) będąca wielokrotnością \(\displaystyle{ p^k}\), że \(\displaystyle{ s(m)=p}\)
kaszubki
[MIX][Teoria liczb] Sumy cyfr

61. Czworokąt
Dany jest czworokąt wypukły \(\displaystyle{ ABCD}\), z jego dwusiecznych zewnętrznych tworzymy nowy czworokąt \(\displaystyle{ PQRS}\). Pokaż, ze jeżeli na \(\displaystyle{ ABCD}\) można opisać okrąg to punkt przecięcia przekątnych \(\displaystyle{ ABCD}\) i środki okręgów opisanych na \(\displaystyle{ ABCD}\) oraz \(\displaystyle{ PQRS}\) są współliniowe.
adamm
[Planimetria] Czworokąty z dwusiecznych zewnętrznych

62. Równanie funkcyjne
Skąd wiadomo, że dla równania funkcyjnego \(\displaystyle{ f(x+yf(x)^{2})^{2}=f(x)^{2}f(y)^{2}}\) zachodzi, że \(\displaystyle{ f(x)^{2}=\sup(1+cx,0)}\)?
pioti1625825
Równania funkcyjne

63. Ograniczony ciąg
Określmy następująco ciąg liczb \(\displaystyle{ p_i}\):
- \(\displaystyle{ p_1, \ p_2}\) są liczbami pierwszymi
- \(\displaystyle{ p_n}\) jest największym dzielnikiem pierwszym liczby \(\displaystyle{ p_{n-1}+p_{n-2}+2000}\)
Udowodnij, że istnieje takie \(\displaystyle{ M}\), że dla każdego \(\displaystyle{ n}\) zachodzi \(\displaystyle{ p_n \le M}\)
Swistak
[Teoria liczb] Ograniczony ciąg liczb pierwszych

64. Funkcja
Powiedzmy, iż mam funkcję rzeczywistą o wartościach rzeczywistych. Czy może być tak, że jest ona ciągła na pewnym zbiorze nigdziegęstym (nieprzeliczalnym) ?
Ao_no_Tengu
Czy funkcja f: R -> R może być ciągła na zb. nigdziegęstym?

65. Funkcja Okresowa
Niech \(\displaystyle{ f: \RR \to \RR}\) będzie taką, że \(\displaystyle{ f(x-1) - f(x+1) = \lfloor f(x)-f(x+1) \rfloor}\) dla \(\displaystyle{ x \in \RR}\). Udowodnić, że \(\displaystyle{ f}\) jest okresowa.
mol_ksiazkowy
Jeszcze raz okresowość
Problem rozwiązany połowicznie przez użytkownika jutrvy.
66. Malowanie kwadratu
Ile jest istotnie różnych pokolorowań kwadratu \(\displaystyle{ 3 \times 3}\), przy których jest 7 pół czarnych i dwa białe?
wskazówka: skorzystaj z lematu Burnside’a
paulina95
lemat Burnside’a

67. Mix Kartka z średniotrudnymi ; Problem 15
Niech \(\displaystyle{ A_n}\) będzie zbiorem wszystkich liczb w formie \(\displaystyle{ 1+ \frac{a_1}{\sqrt{2}} + \frac{a_2}{\sqrt{2}^2}+ …+ \frac{a_n}{\sqrt{2}^n}}\) gdzie \(\displaystyle{ a_j \in \{ -1, 1 \}}\) dla \(\displaystyle{ j =1, …, n}\). Wyznaczyć \(\displaystyle{ \sum_{x , y \in A_n \ x \neq y} xy}\)
mol_ksiazkowy
[MIX] Kartka z średniotrudnymi

68. Wielokrotność
Ułóż dwie liczby wykorzystując wszystkie cyfry: 2,2,3,3,4,4,5,5,6,6,7,7,8,8,9,9 - tak, żeby jedna była dwa razy większa od drugiej.
podarek9
dwie liczby
Problem rozwiązany przez użytkownika Marcin7Cd.
69. Jaki współczynnik ?
Jaki jest współczynnik przy \(\displaystyle{ x^2}\) w rozwinięciu wyrażenia \(\displaystyle{ (x-1)(x-2)(x-3)...(x-n)}\) ?
mol_ksiazkowy
Jaki tu współczynnik?
Problem rozwiązany przez użytkownika pi0tras.
70. Własność \(\displaystyle{ \sqrt{2}}\)
Dany jest ciąg \(\displaystyle{ \lfloor n \sqrt{2} \rfloor = \lfloor \sqrt{2n^2} \rfloor}\), Jego wyrazy są w pierwszym wierszu, a pod nim te których brak tj.
\(\displaystyle{ 1, \ 2, \ 4, \ 5, \ 7, \ 8, \ 9 , ...}\)
\(\displaystyle{ 3, \ 6, 10 , 13, 17 , 20 , 23, ...}\)
Udowodnić, że różnica liczb na \(\displaystyle{ n}\) tym miejscu (w \(\displaystyle{ n}\) tej kolumnie ) to \(\displaystyle{ 2n}\).
mol_ksiazkowy
Własność pierwiastka z dwóch

71. Twierdzenie o trójliściu
W trójkącie \(\displaystyle{ ABC}\) dwusieczna kąta \(\displaystyle{ A}\) przecina okrąg opisany na tym trójkącie w punkcie \(\displaystyle{ M}\). Punkt \(\displaystyle{ S}\) jest środkiem okręgu wpisanego w trójkąt \(\displaystyle{ ABC}\). Udowodnij, że odcinki \(\displaystyle{ MS, MB}\) i \(\displaystyle{ MC}\) mają tę samą długość.
malwinka1058
dwusieczna kąta w trójkącie i okręgi

72. Rozwiązanie nierówności
Znaleźć wszystkie \(\displaystyle{ x,y \in \mathbb N}\) spełnające nierówności \(\displaystyle{ \sqrt{x(\ln x +\ln \ln x)}-1 > y > \sqrt{x(\ln x+ \ln \ln x-1)}}\).
rochaj
[Nierówności] rozwiązanie nierówności

73. Odcinki w trójkącie
Niech \(\displaystyle{ D}\) będzie punktem na boku \(\displaystyle{ BC}\) trójkąta \(\displaystyle{ ABC}\) tak że \(\displaystyle{ AD>BC}\). Punkt \(\displaystyle{ E}\) na \(\displaystyle{ CA}\) taki że \(\displaystyle{ \frac{AE}{EC}=\frac{BD}{AD-BC}}\). Pokaż że \(\displaystyle{ AD>BE}\) .
rochaj
[Planimetria] odcinki w trójkacie

74. Ciekawa suma cyfr
Udowodnić, że w każdym systemie k-kowym, gdzie \(\displaystyle{ k>1}\) dla każdej liczby \(\displaystyle{ a}\) całkowitej dodatniej i względnie pierwszej z \(\displaystyle{ k-1}\) istnieje takie \(\displaystyle{ m}\), że każda liczba całkowita większa od \(\displaystyle{ m}\) jest sumą cyfr jakiejś wielokrotności \(\displaystyle{ a}\) (w tym systemie).
jakub_jabulko
[Teoria liczb] Ciekawa suma cyfr

75. Objętość części wspólnej
Niech \(\displaystyle{ SABC}\) będzie czworościanem o objętości \(\displaystyle{ 1}\), \(\displaystyle{ G}\) -środek ciężkości trójkata \(\displaystyle{ ABC}\), \(\displaystyle{ O}\) - środek odcinka \(\displaystyle{ SG}\). Niech \(\displaystyle{ S'A'B'C'}\) będzie obrazem w symetrii środkowej \(\displaystyle{ SABC}\) względem \(\displaystyle{ O}\). Oblicz objętość części wspólnej \(\displaystyle{ SABC}\) oraz \(\displaystyle{ S'A'B'C'}\).
rochaj
[Stereometria] objętość czesci wspólnej 2 czworościanów

76. Inna funkcja okresowa
Udowodnić, że funkcja spełniająca dla każdego \(\displaystyle{ x\in \RR}\) warunek \(\displaystyle{ f(x+1)= \frac{1+f(x)}{1-f(x)}}\) jest okresowa.
Rabbitvon
Czy podana funkcja jest okresowa?
Problem rozwiązany przez użytkownika mol_ksiazkowy.
77. Zmiany w trójce
Mamy \(\displaystyle{ X= \{ 3, 4, 12 \}}\) i jeśli jakieś \(\displaystyle{ a , b \in X}\) to można je zamienić na \(\displaystyle{ 0,6a -0,8b}\) i \(\displaystyle{ 0,8a +0,6b}\). Czy można zamienić przez takie zamiany \(\displaystyle{ X}\) na zbiór \(\displaystyle{ Y= \{ 4, 6, 12 \}}\) ?
Ukryta treść:    
mol_ksiazkowy
Zamiany w trójce
Problem rozwiązany przez użytkownika Marcin7Cd.
78. Podzielność w liczbach naturalnych
Znajdź wszystkie liczby naturalne \(\displaystyle{ a, b}\) takie że oba wyrażenia \(\displaystyle{ \frac{a^{3}+b}{b^{3}-3a}}\) oraz \(\displaystyle{ \frac {b^{3}+a}{a^{3}-3b }}\) są naturalne
robin5hood
podzielność w liczbach naturalnych

79. Kwadrat liczby
Znależć wszystkie liczby naturalne \(\displaystyle{ n}\) takie że \(\displaystyle{ 2(n^2+n-1)^2+7}\) jest kwdratem liczby naturalnej.
Wiadomo że 1 i 4 pasują ale czy są jeszcze inne a może już nie ma ?
alfred0
kwadrat liczby

80. Suma liczbą trójkątną
Niech \(\displaystyle{ a\ge 1}\) będzie liczbą naturalną.
Oznaczmy \(\displaystyle{ S(n)=1+a+a^2+a^3+...+a^n}\) dla \(\displaystyle{ n=1,2,3,...}\)
Udowodnij, że jeśli \(\displaystyle{ S(n)}\) jest liczbą trójkątna dla każdego \(\displaystyle{ n \in Z_{+}}\), to \(\displaystyle{ a=9}\)
Marcin7Cd
[Teoria Liczb] suma ciągu geometrycznego

81. Całkowite potęgi
Czy istnieje taka liczba nienaturalna \(\displaystyle{ x}\), że \(\displaystyle{ 2^{x}, 3^{x}}\) są naturalne?
ElEski
[Teoria liczb] Nudne i łatwe

82. Siódemka z teorii liczb; układ równań
Udowodnij, że układ
\(\displaystyle{ \begin{cases}x^{6}+x^{3}+x^{3}y+y=147^{157} \\
x^{3}+x^{3}y+y^{2}+y+z^{9}=157^{147} \end{cases}}\)

nie ma rozwiązań w liczbach całkowitych \(\displaystyle{ x, y, z}\).
Elayne
[Teoria liczb] Siódemka z Teoria liczb

83. Dwa wielomiany zespolone
Niech \(\displaystyle{ f, g}\) będą niestałymi wielomianami o zespolonych współczynnikach. Załóżmy, że \(\displaystyle{ f^{-1}(k) = g^{-1}(k)}\) dla \(\displaystyle{ k = 0, 1}\). Czy \(\displaystyle{ f \equiv g}\)?
Takahashi
Dwa wielomiany o wspólnych miejscach zerowych
ODPOWIEDZ